Categories
Carnegie Institute of Technology Columbia Curriculum M.I.T. Pennsylvania

Pennsylvania. Memos from Ando and Dhrymes to the curriculum committee, 1965

 

The significance for the history of economics of the following three memos is that they provide an illustration of the diffusion (infiltration?) of the M.I.T. canon to other departments. Albert Ando taught a few years at M.I.T. before coming to Penn and Phoebus Dhrymes (M.I.T., Ph.D., 1961) wrote his dissertation under Kuh and Solow.  The memos were sent to the curriculum committee of the department of economics at the University of Pennsylvania in January 1965 (at least the Ando memo is dated January 14, 1965 and it explicitly refers to the Phoebus memo and their recommendations to the Mathematics Committee that are undated).

Obituaries for both Ando and Dhrymes have been added to this post and precede the three memos.

Economics in the Rear-view Mirror thanks Juan C. A. Acosta who found these memos in the Lawrence Klein Papers at the Duke University Economists’ Papers Project and has graciously shared them for transcription here. 

Addition to post: At Banca d’Italia, N. 7 – Albert Ando: a bibliography of his writings.

_______________________________

Albert Keinosuke Ando
1929-2002
Obituary

Dr. Albert Ando, professor of economics, SAS and professor of finance, Wharton, died on September 19 [2002] at the age of 72.

Dr. Ando was born in Tokyo, Japan in 1929 and came to the United States after World War II. He received his B.S. in economics from the University of Seattle in 1951, his M.A. in economics from St. Louis University in 1953, and an M.S. in economics in 1956 and a Ph.D. in mathematical economics in 1959 from Carnegie Institute of Technology (now Carnegie Mellon University). Dr. Ando came to Penn in 1963 as an associate professor of economics and finance and became professor of economics and finance in 1967. He held this position until his death.

Dr. Lawrence Klein, Nobel laureate in economics and professor emeritus of economics wrote the following about his colleague.

After World War II many Japanese scholars visited the United States for general education and to modernize their training in some key subjects. Albert Ando, Professor of Economics and Finance, who died of Leukemia last week was an early arrival in the 1940s. He was educated at Seattle and St. Louis Universities and often expressed gratitude at the career start provided by his Jesuit teachers in an adopted country.

He completed the doctoral program in mathematical economics at the Carnegie Institute of Technology, where he was strongly influenced by Herbert Simon with whom he collaborated in research papers on aggregation and causation in economic systems. He also worked closely with another (Nobel Laureate to be) Franco Modigliani on the life cycle analysis of saving, spending, and income.

Dr. Ando was on the faculties of the Carnegie and of the Massachusetts Institutes of Technology before moving to the University of Pennsylvania, where he remained since 1963. He had visiting appointments at universities in Louvain, Bonn, and Stockholm. He consulted with the International Monetary Fund, the Federal Reserve Board, The Bank of Italy, and the Economic Planning Agency of Japan. He held many positions as an editor of scholarly journals and wrote numerous articles and books.

The main contributions of Professor Ando were in econometrics (theory and applications), monetary analysis, demographic aspects of household economic behavior, economic growth, and economic stabilization. His work on the Massachusetts Institute of Technology, University of Pennsylvania, and Social Science Research Council (MPS) model was of great benefit for the research department of the Federal Reserve Board, and his more recent work on econometrics for the Bank of Italy had been very fruitful.

He served as chairman of the graduate group in the economics department, 1986-1989, and developed excellent working relationships with many advanced students. He set very high standards, and those he worked with as thesis supervisor benefited greatly. He was extremely loyal and dedicated to their work, maintaining close connection with them after they departed from the University.

During his long and fruitful career, he earned many honors–as Fellow of the Econometric Society, as a Ford Foundation Faculty Research Fellow; as a Guggenheim Fellow, and a Japan Foundation Fellow. He was given the Alexander von Humboldt Award for Senior American Scientists.

Albert Ando is survived by his wife of 35 years, Faith H. Ando, two professorial sons, Matthew and Clifford, and a daughter, Alison, who has just been admitted to the New York Bar. His mother, sister, and brother, live in Japan.

–Lawrence Klein, Professor Emeritus of Economics

Source: University of Pennsylvania. Almanac. Vol. 49, No. 6, October 1, 2002.

_______________________________

Phoebus James Dhrymes
(1932-2016)

Phoebus J. Dhrymes (1932-2016), the Edwin W. Rickert Professor Emeritus of Economics, was a Cypriot American econometrician who made substantial methodological contributions to econometric theory.  Born in the Republic of Cyprus in 1932, Phoebus Dhrymes arrived in the United States in 1951, settling with relatives in New York City. After a few months, he volunteered to be drafted into the US Army for a two-year tour of duty; afterwards he attended the University of Texas at Austin on the GI Bill. In 1961 he earned his Ph.D. from the Massachusetts Institute of Technology under the supervision of Edwin Kuh and Robert Solow (Nobel Laureate 1987).  After a year-long post-doctoral fellowship at Stanford, he began his professorial career at Harvard, then moved to the University of Pennsylvania, and then UCLA.  In1973 he joined the Department of Economics at Columbia University; he was named the Edwin W. Rickert Professor of Economics in 2003 and retired in 2013.

Econometrics refers to that aspect of the economist’s work concerned with quantifying and testing economic trends. Phoebus Dhrymes‘early research focused on problems of production and investment, but he soon turned to more methodological work and produced important results on time series and on simultaneous equations.  Throughout his career, Phoebus Dhrymes placed much emphasis on the dissemination of scientific knowledge. In the early 1970s he helped found the Journal of Econometrics, which has become the leading journal in this field.  He was also on the advisory board of the Econometric Theory, and was managing editor and editor of the International Economic Review.He was a fellow of the Econometric Society and the American Statistical Association.Dr. Dhrymes was also one of the founders of the University of Cyprus, from which he was later awarded an honorary degree.

He wrote a series of influential textbooks including Distributed Lags:  Problems of Estimation and Formulation. This work was translated into Russian and published by the Academy of Sciences of the Soviet Union, and in the 1970s Dr. Dhrymes was invited to visit the (now former) Soviet Union, specifically Moscow and Novosibirsk. At the time such visits were unusual events for westerners, requiring rarely-issued visas and security clearances, particularly for centers of research such as Novosibirsk.

In a 1999 interview he characterized his books as “filters that distill and synthesize the wisdom of many contributors to the subject.   On this score, I was influenced in my writing by the way I learn when studying by myself.”  (Econometric Theory, 18, 2002)

Dr. Dhrymes is survived by his daughter, Alexis, and his sons, Phoebus and Philip. In his personal life, he was regarded as a generous, kind and gentle man, always there for his family. He came from humble beginnings, and garnered great respect from his family and friends for his achievements. He spoke often of how much he enjoyed teaching. He was always available to his students.He encouraged individualized thinking and understanding of processes rather than rote memorization in learning. He had a warm and affable demeanor, recalled fondly by former students and family members. He will be sadly missed.

Source: Obituary for Phoebus J. Dhrymes at the Columbia University Department of Economics Website.

_______________________________

Memorandum

To: Herbert Levine, Chairman, Curriculum Committee
From: Albert Ando
Subject: Offerings and Requirements in Macroeconomics, Monetary Theory, and Related areas in General Economics Ph.D. Program

  1. Macroeconomics

Enclosed herein is a copy of the outline and references of Economics 621 [The outline and references will be posted later] as I am offering it this fall. It is fairly similar to [the] one year course in macroeconomics which is required of all Ph.D. students at MIT. I am sure that opinions would vary on details, but it is my view that this represents more or less the topics and literature that all Ph.D. students in economics should be familiar with. Ideally, I think there should be another major topic at the end of the outline dealing with current problems and policies.

It is fairly clear that this outline could not be covered in one term, particularly under our present system in which there are only 13 to 14 weeks of classes for a term. As a matter of fact, this fall, with a great deal of rushing throughout the term, I will be able to finish the static part of the outline by the end of the fall term, but certainly no further.

This suggests that the required macroeconomics for Ph.D. students should be two term sequence of courses, the first term dealing essentially with the Keynesian static analysis, and the second term with dynamics, i.e., business cycles and growth models.

  1. Monetary Economics

I have just discovered that Economics 622 is taught without any prerequisite, and that there will be some students in 622 who have not had any macroeconomic theory this spring. I am somewhat stunned, and do not see how I will be able to teach a satisfactory course under the circumstances. This situation is indicated by the fact that 622 is required not only of Ph.D. students in economics but also of master’s candidates, and therefore it is apparently impossible to exclude the students from 622 who have not had 621. An obvious temporary solution is to make those students who have not had 621 wait until next year to take 622. In my view, elements of monetary problems should be included in the first term of the required macroeconomics course, and courses in monetary theory should be made elective. The course in monetary theory should then be taught assuming that students have had adequate preparation in macroeconomics and microeconomics, particularly the theory of general equilibrium, at the level where we can discuss the research and developments in the past dozen years or so, bringing students up to a point where they can draw a thesis topic from their work in the course. There is a room for an argument that there should be another course in addition to the advanced theory course, which deals with more traditional money and banking material. As a matter of fact, I offered two courses in monetary economics at MIT for several years, one dealing with traditional money and banking material taking the one term each of macro and micro economics as prerequisites, and another highly theoretical and advanced course taking two terms each of macro [and] micro economics as prerequisites. It seems to me, however, that Economics 639, Monetary Problems and Policies, should serve as the good traditional money and banking course, so that only one additional course seems to be needed.

  1. Microeconomics and Mathematics

After some discussion with Dhrymes, it is fairly clear that microeconomics should also be taught as a two term sequence. A possible division between two terms would be to deal with partial equilibrium analysis of consumers and firms during the first term, and with the general equilibrium analysis and welfare economics in the second term.

During this fall term, Dhrymes and I found it necessary to conduct a few special remedial sessions in mathematics so that some rudimentary notions of calculus and linear transformation will be available in the discussions in theory courses. The idea, of course, is to arrange so that all students are equipped with minimum of mathematics by the beginning of the second term. If the recommendation of the committee on mathematics is adopted, so that students will learn elementary calculus and the matrices and linear transformation, including rudiments of linear differences and differential equations at the level suggested by the committee it is possible to synchronize it with theory courses so that theory courses will be using only those mathematics students are learning in mathematics remedial courses. For instance, the first term of macro theory would not require too much mathematics except the notion of the systems of equations and their solutions, and the first term of micro theory not much more than the condition of extremum in a fairly informal manner. In the second term, on the other hand, theory courses will require conditions of stability in the general equilibrium analysis, and the difference and differential equations in dynamic models in macroeconomics.

  1. Overall First year program and Second year fields of specialization.

In addition to micro and macro theories and mathematics required for these theory courses, students should be asked to learn minimum of statistics and econometrics. The level of statistics and econometrics should be maintained at the level of text books such as Frazer, Brunk, or Mood plus Johnston.

The implication of the above statement is that the course schedule for typical first year Ph.D. students should look as follows:

First term:

Microeconomics I (Partial equilibrium analysis)
Macroeconomics I (Static Keynesian analysis, including some monetary considerations).
Mathematics I (Elementary calculus)*
Mathematics II (Elementary Linear Algebra)*
Economic History (For those with Adequate mathematical training)

*For the suggested content of mathematics courses, see recommendations of Mathematics Committee.

Second Term:

Microeconomics II (General equilibrium analysis and welfare economics).
Macroeconomics II (Dynamics, business cycles and growth)
Econometrics (6 hour course)

This schedule, of course, would be subject to variations depending on the background and preparations of students. For instance, students who already have sufficient mathematical training might be encouraged to take a course in economic history and a course in somewhat more advanced mathematics, such as mathematical theory of probability or a course in topology in the first term in place of Mathematics I and II.

_______________________________

Lists of Topics for Mathematics for Economists
[Recommendations of Ando and Dhrymes submitted to the Mathematics Committee]

(Mr. Balinski is to suggest some alternative text books)

  1. Calculus
    1. Sets and Functions.
      1. Definitions
      2. Operations on Sets and Subsets.
      3. Relations, Functions.
        K.M.S.T. Chapter 2, Sections 1 through 6, possibly Sections 10 through 13.
    2. Functions, Limits, and Continuity.
    3. Differentiation and Integration of Functions of one variable.
      1. Concepts and Mechanics.
      2. Infinite series and Taylor’s Theories.
      3. Extremum Problems.
    4. Differentiation and Integration of Functions of many variables.
      1. Concepts and mechanics.
      2. Extremum problems, nonconstrained and constrained.
      3. Implicit Function Theorem.
        Any elementary text book in Calculus (e.g. Thomas; Sherwood and Taylor), Supplemented by some sections of a slightly more advanced text on Implicit Function Theorem and La Grange multipliers.
  2. Linear Algebra and others.
    1. Vector Spaces and Matrices.
      1. Vector Spaces and Matrices, Definitions, and Motivations.
        Perlis, Chapters 1 and 2.
      2. Linear Transformations.
        K.M.S.T., Chapter 4, Sections 7 through 12.
      3. Equivalence, Rank, and Inverse.
        Perlis, Chapter 3.
        Perlis, Chapter 4.
      4. Quadratic Forms, Positive Definite and semi-definite Matrices.
        Perlis, Chapter 5, Sections 1, 2, and 5
      5. Characteristic Vectors and Roots.
        Perlis, Chapter 8, Sections 1 and w[?], Chapter 9, Sections 1, 2, 5, and 6.
      6. Difference and Differential Equations; Linear with Constant Coefficients.
        Goldberg, Chapters 1, w, e, and Chapter 4, Sections 1 and 5; Perlis, Chapter 7, Section 10. Some reference to two dimensional phase diagram analysis of non-linear differential equations with 2 variables. Lotke?
      7. Convex Sets.
        K.M.S.T., Chapter 5.

_______________________________

MEMORANDUM
January 14, 1965

To: Curriculum Committee
From: Phoebus J. Dhrymes
Subject: Mathematics, Microeconomics, Statistics and Econometrics in the Economics Graduate Training Program

  1. Mathematics

It has become quite apparent to me during the course of the last term that our students are woefully equipped to handle instruction involving even very modest and elementary mathematics.

I think it is quite generally accepted that a student specializing in Theory, Econometrics and to a lesser extent International Trade and Industrial Organization would find it increasingly difficult to operate as a professional economist, and indeed seriously handicapped in satisfactorily carrying on a graduate study progress, without adequate mathematical training. With this in mind Albert Ando and I have prepared a tentative list of topics that graduate students ought be minimally familiar with and which has been presented to the Mathematics Committee.

This could form a remedial (and a bit beyond) course to extend over a year and to be taken (by requirement or suggestion) by students intending to specialize in the fields mentioned above during their first year of residence.

  1. Microeconomics

It has been my experience in teaching Econ. 620 that one semester is a rather brief period for covering the range of microeconomic theory a graduate student in Pennsylvania ought to be exposed to. As it is the case at both Harvard and MIT, I would propose that the course Econ. 620 be extended to a year course. Roughly speaking, the topics to be covered might be:

  1. Theory of Consumer Behavior
    1. the Hicksian version
    2. the von Neumann-Morgenstern version, including the Friedman-Savage paper
  2. Demand functions, elasticities, etc.
  3. Theory of the firm; output and price determination
    1. Production functions
    2. Cost functions and their relations to i.
    3. Revenue and profit functions and the profit maximizing hypothesis
    4. The perfectly competitive firm and industry, and their equilibrium; comparative statics; supply functions
    5. The monopolistic firm
    6. Monopolistic competition
    7. Duopoly and oligopoly
  4. Factor employment equilibrium
    1. Factor demand functions
    2. Factor employment equilibrium under various market institutional arrangements
    3. Some income distribution theory
    4. Factor supply.
  5. General Equilibrium Analysis; Input-Output models
  6. Welfare Economics (Samuelson; Graaf)
  7. Capital Theory (Fisher, Wicksell, recent contributions)
  8. (Marginally) Some revealed preference theory; or neoclassical growth models; or alternative theories of the firm (e.g., Cyert and Marsh)

It would be desirable if students were sufficiently well-equipped mathematically to handle these topics at some level intermediate between Friedman’s Price Theory Text and Henderson and Quandt; however, since this is not the case at present some other alternative must be found, such as in the manner in which the propose mathematics course is taught, and the order in which topics above are covered. The split of the subjects could be a) through c) or d) for the first semester and the remainder for the second semester. Clearly, neither the topics proposed nor the split represent my immutable opinion and there is considerable room for discussion.

  1. Statistics

At present the statistical training of our students suffers from their inadequate mathematical preparations.

It is my opinion that minimally we should require of our students that they be familiar with the elementary notions of statistical inference, estimation, testing of hypotheses and regression analysis at the level of, say, Hoel, or Mood and Graybill, or any other similar text, (a semester course). For students intending to specialize in Econometrics or other heavily quantitative fields, then it should be highly desirable that a year course be available, say at the level of Mood and Graybill, Graybill, or Fraser, Hogg and Craig, Brunk, etc., with suitable supplementary material. Since, we do have access to a statistics department it might be desirable for our students to take a suitable course there.

Again, due to the problems posed by the mathematics deficiency of incoming students, some accommodation must be reached on this score as well.

  1. Econometrics

Econometrics should not be a required subject; rather the requirement—minimal requisite—should be confined to the one semester course indicated under III. It would be desirable to offer a year course to be taken after the statistics sequence and which would cover at the level of, say, Klein, Goldberger, or my readings showing applications and problems connected thereto.

Topics, could start by reviewing the general linear model, Aitken estimators and similar related topics; simultaneous equation and identification problems, k-class estimators, 3SLS, maximum likelihood estimation, full and limited information, Monte Carlo methods.

Also selected topics from Multivariate Analysis; specification analysis, error in variable problems; elements of stochastic processes theory and spectral and cross spectra analysis.

It might be desirable to teach these subjects in the order cited above, although it would appear preferable to have multivariate analysis precede the review of the general linear model.

  1. General Comments:

I generally agree with Albert Ando’s memorandum on proposed curriculum revision in so far as they pertain to Mathematics requirements, Macro-economics and Monetary Theory.

I think that at present we require our students to take too many courses. I would favor only the following requirements; the basic Micro and Macro year courses. At least a semester of statistics, as indicated under III, and one semester in either economic history or history of economic thought—although I do not feel too strongly on the latter. I presume, in all of this that students in our program are only those ultimately aiming at specialization in Theory, Econometrics, International Trade, Industrial Organization, and possibly Comparative Systems, or Soviet Economics. It is my understanding that our curriculum will not cover those concentrating in Labor Relations, Regional Science or Economic History.

Thus, through their first year our students would be taking more or less required courses, with the second year essentially left open for their special fields of concentration.

Thus, the course program of a typical first year student will look more or less as shown in Albert Ando’s memorandum, p. 4, although I would be somewhat uneasy about requiring 6 hours of mathematics in the first term and 6 hours of statistics (econometrics) in the second term of the first year. Nonetheless I do not object strongly to this, and indeed in this past term many of the students taking 620 and 621 had in effect taken a six-hour course in Mathematics, 611 as taught by Dorothy Brady and approximately 3 hours as taught by Albert Ando and myself.

Quite clearly the above are merely proposals intended to serve as a basis for discussion an ultimately for guidance of entering students in planning their program of study rather than rigid requirements.

 

Source: Duke University, David M. Rubenstein Rare Book and Manuscript Library. Economists’ Papers Archive, Lawrence Klein Papers, Box 19, Folder “Curriculum”.

Images: Left, Albert Ando; Right, Phoebus Dhrymes. From the respective obituaries above.

Categories
Exam Questions Gender Harvard Radcliffe Socialism Suggested Reading

Harvard. Exams and reading period assignment for Programs of Social Reconstruction (Socialism). Mason, 1933.

 

In the collection of final examinations in the Harvard archives, I came across both the Radcliffe and Harvard final examinations for the identical course with the title “Programs of Social Reconstruction” taught by Edward S. Mason. This course was one of the undergraduate staples offered earlier by Thomas Nixon Carver that was handed off to Mason starting 1926/27. 

A few things I find interesting from the materials I was able to find for this year (Note: a course reading list for 1928 needs some work, will be posted later):

  • The final examination questions only cover Marxian socialist theory and movements except for the question  on the reading period assignment that is dedicated to contemporary U.S./U.K. reform. It is possible that earlier utopian socialist literature, Henry George, and anarchism were tested in a mid-term examination, or of course the course description had not been changed. The exact same course description was used by Mason for the 1928-29 academic year.
  • From the Harvard President’s report and the final exam (note the superscript “1” which means first term), it would appear that Mason taught the course in the first term of 1932-33 and not during the second term as announced earlier in the Harvard Register. So it does appear that he taught the course one semester to Harvard men and the following semester to Radcliffe women, so having different final examinations makes sense.
  • The Harvard exam as printed can be compared to the Radcliffe exam to see that there is an obvious type:  the first question only be allocated one hour and the remaining four questions would fill the rest of the examination time.

____________________

Radcliffe College Course Announcement

Economics 7c 2hf. Programs of Social Reconstruction

Half-course (second half-year). Tu., Th., and (at the pleasure of the instructor) Sat., at 9. Asst. Professor E. S. Mason.

 

Source: Radcliffe College. Courses of Instruction, 1932-33. Page 87.

____________________

Harvard Course Announcement with Course Description

Economics 7c 2hf. Programmes of Social Reconstruction

Half-course (second half-year). Mon., Wed., and (at the pleasure of the instructor) Fri., at 10. Associate Professor Mason.

A comparison of the various radical programmes, such as socialism, communism, anarchism and the single tax, the theories upon which they are based, and the grounds of their attack upon the present industrial system. An examination of the various criteria of distributive justice, and of the social utility of the institution of property. A comparison of the merits of liberalism and authoritarianism, of radicalism and conservatism. An analysis also of the present tendencies toward equality under liberalism in this country.

 

Source: Division of History, Government, and Economics, 1932-33 in Official Register of Harvard University, Vol. XXIX, No. 32 (June 27, 1932), p. 74.

____________________

Course Enrollment (Harvard)

[Economics] 7c 1hf. Associate Professor Mason.—Programs of Social Reconstruction.

Total 42: 26 Seniors, 10 Juniors, 2 Sophomores, 4 Others.

 

Source: Report of the President of Harvard College, 1932-33, p. 65.

____________________

Reading Period Assignment

Economics 7c

Read one:

1. Norman Thomas, America’s Way Out.
2. Stuart Chase, A New Deal.
3. George Soule, A Planned Society.
4. Sidney and Beatrice Webb, A Constitution for the Socialist Commonwealth of Great Britain.

Source: Harvard University Archives. Syllabi, course outlines and reading lists in economics, 1895-2003. Box 2, Folder “Economics, 1932-1933”.

____________________

1932-33
RADCLIFFE COLLEGE

ECONOMICS 7c
Final Examination

I

Allow about one hour.

  1. Write a critical review of the book you read for the reading period.

II

Answer four of the following questions.

  1. What position does technological change occupy in Marx’s theory of the decline of capitalism?
  2. What importance has economic imperialism for the tactics of a socialist party according to Marxian theorists?
  3. How do you explain the collapse of the Second International in 1914.
  4. Discuss the validity of the labor-hour as a unit of cost in a socialist planned economy.
  5. Can Marx’s theory of value be reconciled with his explanation of the tendency toward an equal rate of profit in all industries? Discuss.

Final. 1933

 

Source: Harvard University Archives. Harvard University Examination Papers, Finals 1933 (HUC 700028, No. 75). Papers Printed for Final Examinations. History, History of Religions,…Economics,…Military Science, Naval Science. January—June, 1933.

____________________

1932-33
HARVARD UNIVERSITY

ECONOMICS 7c1
Final Examination

Allow about one hour.

  1. Write a review of the book you read for the reading period assignment.
  2. “The essence of the Marxian contribution to socialism was and is the discovery of the proletarian path to power.” Discuss.
  3. What does Lenin mean by economic imperialism?
  4. Consider the position in the history of socialist thought of one of the socialist leaders before Marx.
  5. “With his ‘socially necessary labor time’ Marx anticipated the Technocrats by three quarters of a century and proposed a technological measure of cost and value whose use would immediately put an end to all the stupid absurdities of the price system.” Discuss.

Final. 1933.

Source: Harvard University Archives. Harvard University Examination Papers, Finals 1933 (HUC 700028, No. 75). Papers Printed for Final Examinations. History, History of Religions,…Economics,…Military Science, Naval Science. January—June, 1933.

Image Source:  Edward S. Mason in Harvard Album 1934.

Categories
Chicago Faculty Regulations Texas

Chicago. Policies on faculty paid leave, trips to conferences, 1917

 

 

Regular paid sabbatical leave was not the policy of the University of Chicago at least up through World War I. Also faculty costs to attend professional meetings were not covered unless the faculty member was representing the University in some official capacity.

_____________________

THE UNIVERSITY OF TEXAS
AUSTIN

OFFICE OF
THE DEAN OF THE FACULTY

February 10, 1917

My dear Sir:

It would be of real assistance to us here in defending ourselves against attack if you would give me the practice of your institution on the following points:

  1. Leaves of absence with pay to members of the faculty.
  2. Appropriations to enable members of the faculty to attend meetings of learned societies.
  3. Appropriations for rousing the interest of the alumni.

I should greatly appreciate a prompt reply.

Very truly yours,

[signed, W. J. Battle]
Dean of the Faculty

President Harry P. Judson,
University of Chicago,
Chicago, Ill.

_____________________

Carbon Copy of President Judson’s response to Dean Battle

Chicago, February 15, 1917

Dear Dr. Battle:

Your favor of the 10th inst. is received. In answer to your questions:

  1. Leave of absence with pay to members of the faculty is given rarely, and only for a particular reason. These reasons are in some cases those relating to the health of the individual in question; in other cases relating to a special piece of scientific work to be undertaken.
  2. We make no specific appropriation to enable members of the faculty to attend the meetings of learned societies. Rarely a specific allowance is made in cases of meetings in which members of the faculty are sent to represent the University.
  3. We have some appropriations for the development of close relations between the alumni and the University. Money so spent is, I think, very wisely spent.

You are aware that our plan of organization contemplates four quarters in the year, and that any member of the faculty is expected to give instruction during three out of those four. He is allowed to give instruction during the fourth frequently, and such work may be credited as vacation, so that later the person in question may be absent on full pay for a considerable time.

Very truly yours,

H.P.J.-L

Dean W. J. Battle
The University of Texas
Austin, Texas

 

Source:   University of Chicago Archives. Office of the President. Judson and Burton Administrations, Records. Box 68, Folder 18 “Procedures, 1893-1917”.

Image Source:  Cobb Lecture Hall, 1917. University of Chicago Photographic Archive, apf2-02760, Special Collections Research Center, University of Chicago Library.

Categories
Chicago Economics Programs Fields

Chicago. Memo to Dean from Chair of Economics. Strengths & Weaknesses, 1955

 

What I found particularly striking in the following memo, written by the chairman of the Chicago department of economics in 1955, is the number of fields in which the department saw itself weak or at least in need of support: labor, international, mathematical economics and econometrics, development, and industrial organization. Perhaps this was just a matter of administrative strategy, beg for assistance for five fields and hope to actually get assistance for three. That said, Schultz does not appear to be engaging in three-dimensional chess here. Will be interested in hearing what other people think about the this memo.

_______________________

Carbon Copy of Strengths and Weaknesses Memo
T.W. Schultz to Dean Chancy D. Harris

September 22, 1955

[To:] Dean Chancy D. Harris
[From:] Theodore W. Schultz
[Re:] Department of Economics

 

It may be helpful to have me briefly state the major elements of strength and, also, of weaknesses which I see in economics, in the hope that these notes may serve you as you prepare your presentation for the trustees.

Elements of Strength

  1. A comparatively young faculty strongly committed to research and graduate instruction.
  2. Research and related seminars are effectively organized as small scale enterprises:
    1. Workshop on Money
    2. Workshop on Public Finance
    3. Resources Research Enterprise
    4. Technical Assistance Studies
    5. Studies of Russia Agriculture
    6. Inventory Studies
  3. Satisfactory foundation support for some of the workshops and research enterprises now underway:
    1. Rockefeller Foundation supporting the money and public finance workshops.
    2. Resources for the Future supporting the resources research.
    3. Ford Foundation supporting the technical assistance studies.
    4. Also, for individual research, the Rockefeller Foundation support of economic history of Professor Hamilton.
  4. U. S. Government contracts and grants are proving satisfactory in financing some research:
    1. The inventory studies
    2. Russian agriculture work
  5. Financial support for competent advanced graduate students doing research is available from the several small scale research enterprises and, also, from SSRC (Griliches this year); from Earhart funds (Nerlove); and from corporations (Oi)
  6. Our new Ph.D. theses procedure is proving most effective in bringing student and faculty resources to bear on productive research.
  7. The new Economic Research Center of the Department is now proving important and necessary overhead facilities and services required by faculty and students working in the several small scale research enterprises.
  8. The new arrangements with the University Press to publish our Studies in Economics represents a major advance.
  9. The Journal of Political Economy continues strong as Prof. Rees and Miss Bassett take over.
  10. While we are not satisfied with the “quality” of many of our graduate students, we appear to be holding our own in a period when many averse factors are at work in lowering the quality of students in most branches, and also in economics generally as it appears.

 

Elements of Weakness

  1. Too many of the faculty are now in junior roles and there are too few major staff members on indefinite tenure in view of the fields of specialization in economics, the range and number of advanced graduate students, and the research work that is underway.
  2. With Professor Harbisons’s leaving and the non-functioning of the Industrial Relations Center in economics, our work in labor economics needs to be reorganized and strengthened. This replanning is now underway. Research resources are required: about $20,000 a year would be optimum.
  3. We are not prepared to serve adequately most of the many (representing about 30% of our graduate students) foreign students working in economics:
    1. Many of them should be in a modified Master’s program.
    2. Relevant research should concentrate on “developmental” problems.
    3. More effort is required to guide their work.
  4. The Department is now weak in International Economics because of the illness of Professor Metzler.
  5. The work in consumption economics has not been made as effective as it should be in bringing major graduate students into play in research.
  6. The reorganization and staffing of work in Mathematical Economics and Econometrics, with the Cowles Commission leaving, is unfinished business:
    1. Professor Hans [a.k.a., “Henri”] Theil is here this year as visiting Professor.
    2. Plans beyond this year await action.
    3. No research support at present for advanced students or for complementary staff in this important area.
  7. The broad area of Economic Development requires major attention and it should be placed high on our agenda as we develop plans and staff during the next few years:
    1. This area is needed to serve especially graduate students from foreign countries.
    2. The economic problems are important to the U.S. scene also.
    3. The Research Center for Economic Development and Cultural Change and importantly the “Journal” it has established need to be drawn into this new effort.
    4. Major new research resources are required.
  8. The long neglected field of Industrial Organization.

 

Some Concrete Steps

  1. To establish the work in Mathematical Economics about $20,000 a year will be required for a “professor” to head this work, for complementary staff, and related research.
  2. To establish the new enterprise now contemplated in Economic Development about $50,000 a year appears essential.
    In this area, a professorship, a visiting professor for each of the next several years, complementary staff, student research in a workshop and support for the Journal “Economic Development and Cultural Change.”
  3. Also in Labor Economics we need to move to a professorship and research support of about $20,000 a year.
  4. How to strengthen the work in International Economics must await developments affecting Professor Metzler’s recovery.
  5. There remains then the long neglected area usually referred to as Industrial Organization. Since no major individual has emerged here or elsewhere, we are compelled to “invest” in a younger person in breaking into this area.

 

Source:   University of Chicago Archives. Department of Economics Records. Box 42, Folder 8.

Image Source:  T. W. Schultz, University of Chicago Photographic Archive, apf1-07484, Special Collections Research Center, University of Chicago Library.

Categories
Gender Harvard Michigan

Harvard. Ph.D. Alumna (1951). Michigan Professor Eva Mueller. 1920-2006

 

One probably would have forecast that Eva L. Mueller who was awarded her economics Ph.D. (Radcliffe College) in 1951 with the dissertation “Business Savings and the Business Cycle” would have gone on to become a macroeconomist. Arthur Smithies was the chairman of the Harvard economics department at the time she received her Ph.D. and when asked for his help in finding a job, Eva Mueller remembered him saying “…he couldn’t help me, since economics wasn’t a woman’s field”. She did find a job at the University of Michigan’s Institute for Social Research where she transformed herself into a population and development expert.

I have copy-and-pasted a variety of biographical memoirs and obituaries for this post. One cannot help but be touched by the deep affection and respect of her Michigan colleagues that one can read in and between the lines. Macro’s loss was development economics’ gain.

_____________________

Professor Eva Mueller, 1920-2006

Professor Eva Mueller died on November 19, 2006 at the age of 86.

Dr. Mueller, a U-M Professor Emerita of Economics and Research Scientist at PSC at the time of her death, had a long and fruitful career at Michigan. But she faced many challenges on her path to success.

Fifty-five years ago, when Mueller received a PhD in economics from Harvard University, she asked the chairman of the department for help finding a job. “He said he couldn’t help me, since economics wasn’t a woman’s field,” she recalled. Undeterred, Mueller found a research job at the Institute for Social Research (ISR), where she helped to pioneer the use of surveys to analyze consumer behavior. After six years, she received a tenure-track appointment as an assistant professor in the Department of Economics and in 1964 was named a full professor.

“The struggle isn’t over yet,” she said, accepting the Carolyn Shaw Bell Award Jan. 6, 2001 from the American Economic Association’s Committee on the Status of Women in the Economics Profession.

Mueller was nominated for the award, given annually to an individual who has furthered the status of women in economics, by several former students, along with David Lam, professor of economics and director of ISR’s Population Studies Center, and Sherrie Kossoudji, associate professor of social work and adjunct associate professor of economics.

“Eva was really unusual as a woman breaking into the male-dominated field of economics,” noted Lam. “She was a real role model for many of us,” said Kossoudji. “She was also consistent in her support for young female students. And she made us tough. ‘You must do better,’ she told us. ‘You must work harder.’ That was always her approach.”

Born February 26, 1920, Mueller said she was influenced by the Great Depression in making her career choice. “It impressed me that what the world needed was to rescue its economies,” she said. Also, her mother, who had a PhD in chemistry, emphasized education for her children. “She had her mind set that all her children must get PhDs.”

During WWII, she said, when the Harvard Economics Department “was more or less closed down… because all of the faculty was in Washington working on the war effort,” she took a job at the Federal Reserve Bank in New York.

After the war, Mueller studied under Alvin Hansen, “at the time the number one Keynesian in the U.S.” Upon completing her PhD, she needed a job.

“I wrote some letters to people whose stuff I had read and thought they would be interesting to work for. One was George Katona. He happened to know George Garvey, for whom I had worked at the Federal Reserve, so he wrote to him and asked if I would be a plausible candidate. That’s how I came to Michigan to the Institute for Social Research.

I wanted to go to the economics department, but they would not accept me. Then I was sort of on the waiting list. John Lansing and, I think, even Jim Morgan, were on the waiting list ahead of me. They eventually got to me.”

In 1951 Mueller joined the staff of the Survey Research Center; in 1957 she joined the Department of Economics, where she became a full professor in 1964. Six years later, she became a research scientist at the Population Studies Center. Mueller had been a Professor Emerita since 1988.

During her long and active career at Michigan, Eva Mueller made important contributions in several areas of economic research. For the first two decades, her research emphasized analysis of consumer behavior in the U.S. She later moved on to work related to economic development and economic demography. Her published papers cover a wide range of topics and countries, including the impact of unemployment on consumer confidence in the U.S., the economics of fertility decline in Taiwan, and the time allocation of women and children in Botswana.

In addition to her contributions as a researcher, she played an important role in building the economic demography training program run jointly by the Population Studies Center and the Department of Economics. She served as an advisor to many PhD students in economic demography who have gone on to positions in universities, government, and international agencies.

The Eva Mueller New Directions in Demography and Economics Fund has been established to support research and training in demography and economics, especially projects focusing on low income countries and projects dealing with the socioeconomic position of women and investments in children’s health and human capital.

 

Source: Announcement of the death of Eva L. Mueller by the University of Population Studies Center, Institute for Social Research at the University of Michigan.

_____________________

Select Career Publications

Dr. Mueller studied the interaction of economic and demographic change. One particular focus of her research was the relation between income change and fertility change. Within this context she was interested in the methodology of collecting useful employment statistics, including the methodology of time-use studies.

Journal Articles

Mueller, Eva. 1984. “The Value and Allocation of Time in Rural Botswana.” Journal of Development Economics, 15(1-3): 329-60. Abstract.

Watanabe, B., and Eva Mueller. “A Poverty Profile for Rural Botswana.” World Development, 12, no. 2 (1984): 115-27. Abstract.

Kossoudji, S., and Eva Mueller. “The Economic and Demographic Status of Female-Headed Households in Rural Botswana.” Economic Development and Cultural Change, 31, no. 4 (July 1983): 831-59.

Mueller, Eva. “The Impact of Demographic Factors on Economic Development in Taiwan.” Population and Development Review, 3, no. 1&2 (1977): 1-22. Abstract.

Mueller, Eva, and R. Cohn. “The Relation of Income to Fertility Decisions in Taiwan.” Economic Development and Cultural Change, 25, no. 2 (January 1977): 325-47.

MacDonald, M., and Eva Mueller. “The Measurement of Income in Fertility Surveys in Developing Countries.” Studies in Family Planning, 6, no. 1 (January 1975): 22-28. Abstract.

Mueller, Eva. “Economic Motives for Family Limitation.” Population Studies, 27, no. 3 (November 1972): 383-403. Abstract.

 

Chapters

Mueller, Eva. 1976. “The Economic Value of Children in Peasant Agriculture.” In Population and Development: The Search for Selective Interventions edited by Ronald Gene Ridker. Baltimore : The Johns Hopkins University Press.

 

PSC Reports

Mueller, Eva. “Time Use Studies: Their Potential Contribution to the Policy Dialogue in Developing Countries.” PSC Research Report No. 85-86. 9 1985.

Mueller, Eva, and Kathleen Short. “Income and Wealth as They Affect the Demand for Children in Developing Countries.” PSC Research Report No. 82-35. 9 1981.

Kassoudji, Sherrie, and Eva Mueller. “The Economic and Demographic Status of Female Headed Households in Rural Botswana.” PSC Research Report No. 81-10. 3 1981.

 

Source:  University of Population Studies Center, Institute for Social Research at the University of Michigan.

_____________________

Eva L. Mueller
Memoir
1988

Eva L. Mueller, Professor of Economics and Research Scientist in the Population Studies Center and Center for Research on Economic Development, will retire from active faculty status on December 31, 1988, after a most productive career as a teacher and researcher.

A native of Germany, Professor Mueller became a naturalized citizen in 1944. She received her B.A. degree from Smith College in 1942, her M.A. degree from New York University in 1945, and her Ph.D. degree from Harvard University in 1951.

From 1951-68, Professor Mueller was on the staff of the Survey Research Center at the Institute for Social Research. She joined the Department of Economics in 1957 as an assistant professor; she was promoted to associate professor in 1960 and to professor in 1964. Since 1968, she has also been affiliated with the Center for Research in Economic Development and the Center for South and Southeast Asian Studies, and in 1970, she accepted an additional appointment in the Population Studies Center. From 1974-78, Professor Mueller served as associate dean in the College of Literature, Science, and the Arts.

For several years, Professor Mueller has directed the NIH-sponsored Economic Training Program in Economic Demography, which has attracted many of the brightest students in the Ph.D. program in economics. Several of the department’s most successful female students have been recruited into the program. They were attracted by the setting Professor Mueller created, which was encouraging and supportive, and in which Professor Mueller herself has acted as an extraordinary role model.

Professor Mueller has conducted exciting and important research in the area of fertility and female labor supply in developing countries. Some of her work has been conducted under the auspices of the World Bank and the U.S. Agency for International Development, taking her to India, Thailand, Brazil, and Botswana.

The Regents now salute this distinguished educator and researcher for her dedicated service by naming Eva L. Mueller Professor Emeritus of Economics.

 

Source: Mueller named Emerita Professor at University of Michigan. Faculty History Project.

_____________________

 Obituary
Eva L. Mueller
1920 – 2006

Eva Mueller, Professor Emerita of Economics, died November 19, 2006, in Ann Arbor, at the age of eighty-six.

Professor Mueller received her B.A. in 1942 from Smith College with a major in economics. In 1951 she received her Ph.D. in economics from Harvard University and joined the staff of the University of Michigan’s Survey Research Center. In 1957 she joined the Department of Economics. She became associated with the Center for Research in Economic Development and the Center for South and Southeast Asian Studies in 1968, and joined the Population Studies Center in 1970. Her many roles at the University of Michigan included service as Associate Dean in the College of Literature, Science, and the Arts.

Mueller received a number of distinctions during her career. She was a Fellow of the American Statistical Association. She served on the Board of Directors of the Population Association of America and was elected Vice-President of the Association. In 2001 she received the Carolyn Shaw Bell Award from the American Economics Association. This award is given by the AEA’s Committee on the Status of Women in the Economics Profession “to an individual who has furthered the status of women in the economics profession, through example, achievements, increasing our understanding of how women can advance in the economics profession, or mentoring of others.”

Mueller made important contributions in several areas of economic research. For the first two decades, her research emphasized analysis of consumer behavior in the United States. She later moved into research related to economic development and economic demography. Her published papers cover a wide range of topics and countries, including the impact of unemployment on consumer confidence in the U.S., the economics of fertility decline in Taiwan, and the time allocation of women and children in Botswana. In addition to her contributions as a researcher, she played an important role in building the economic demography training program run jointly by the Population Studies Center and the Department of Economics. She served as an advisor to many Ph.D. students in economic demography who have gone on to positions in universities, government, and international agencies.

—David Lam, Department of Economics

Source: Obituary for Eva L. Mueller, University of Michigan. Faculty History Project.

Image sources:  Early career portrait of Eva L. Mueller from University of Population Studies Center, Institute for Social Research. Later portrait from University of Michigan, Faculty History Project.

Categories
Cambridge Exam Questions

Cambridge. Economics Tripos Papers, 1931.

 

While Economics in the Rear-view Mirror’s ambition is to be the boutique blog of economics education  in the United States up through the 1960s, from time to time I’ll venture off the North American continent to explore English and German departments, seminars etc.  Earlier I have transcribed and posted the 1891 guide to the Cambridge Moral Tripos that preceded the Economics Tripos but did have a Political Economy component. Also I have transcribed and posted the exams from Oxford’s Philosophy, Politics, and Economics (PPE) program for 1931 that I found in Wesley Clair Mitchell’s papers. As serendipity would have it, my trip last year to the Library of Congress yielded several years’ worth of exams from the Cambridge Economics Tripos. Below you will find the examination papers for 1931 that conveniently coincide with the PPE papers from Oxford. 

I recently found that a copy of the 1931-1933 Cambridge Economics Tripos is available at hathitrust.org.

________________________

ECONOMICS TRIPOS
PART I.

GENERAL PRINCIPLES I.
(NEW REGULATIONS.)

MONDAY, JUNE 1, 1931. 9—12.

  1. Explain clearly the meaning of the term Marginal Utility and its relation to the conception of Consumer’s Surplus.
  2. How far can a theoretical distinction be drawn between the rent of a house and the rent of a field? Is the distinction in all cases valid?
  3. “There is no real connection between price and cost of production throughout an industry as a whole.” Discuss.
  4. In what conditions would competition be said to be perfect? How far and for what reasons is competition in practice often imperfect?
  5. Explain in what circumstances it is possible that a commodity will be produced under conditions of Diminishing Returns. Is it conceivable that the same commodity should be produced at one time under Diminishing, at another under Increasing Returns?
  6. “Russia can grow wheat more cheaply than England because all rents have been abolished in Russia, while rents must still be paid in England.” Examine the validity of this argument.
  7. In what sense can it be said that labour has (a) a demand price, (b) a supply price, (c) a cost of production?
  8. What is likely to be the influence upon the rate of interest of improvements of industrial technique?
  9. Explain, if possible with diagrams, what considerations will influence a monopolist in determining the price that he should charge for his product. In what circumstances will the price be considerably higher than the competitive price?
  10. How would you explain the differences (a) between the wages of coal miners and agricultural workers in this country, (b) between the wages of coal miners in this country and coal miners in Poland?

 

ECONOMIC STRUCTURE.
(NEW REGULATIONS.)

MONDAY, JUNE 1, 1931. 1.30—4.30.

  1. What are the advantages and disadvantages of limited liability?
  2. Explain the factors which determine the size of the productive unit in different industries, giving examples.
  3. What are the functions of a merchant, particularly in the export trades? When is it likely to be to the advantage of a British manufacturer to become his own merchant?
  4. “Left to himself, the consumer would never welcome mass production.” Comment.
  5. Classify the relative importance of the different industries in any neighbourhood with which you are acquainted, explaining how they came there and why they stay there.
  6. In what ways do the problems of management set a limit to the size of the unit of control in industry?
  7. Critically examine the argument for and against vertical combination.
  8. Outline proposals for the reorganization in this country of either shipbuilding or cotton manufacture.
  9. Describe the economic functions of the London Stock Exchange.
  10. Figures published by the Ministry of Labour suggest that industrial employment in Great Britain is moving south-eastwards. Can you account for this?
  11. Under what conditions is an industry likely to become a monopoly? Describe briefly methods for the State control of industrial monopoly.

 

GENERAL PRINCIPLES II.
(NEW REGULATIONS.)

TUESDAY, JUNE 2, 1931. 9—12.

A.

  1. “If an increase in cost of production causes the price of a commodity to rise, the resulting fall in demand will lower its price again and cause it to return eventually to its former level.” Point out the confusion involved in this statement, and illustrate your answer if possible by a diagram.
  2. Consider the nature and importance of the distinction between real wages and nominal wages, and between wage rates and total earnings.
  3. What is likely to be the effect of an increased demand for mutton on the prices of (a) mutton, (b) beef, (c) wool, (d) the wages of woolen workers? Give reasons in each case for your views.
  4. What are the chief characteristics of the English system of land tenure? What do you consider to be the main advantages and disadvantages of this form of tenure?
  5. What causes determine the normal rate of interest in England? What would be the effects of a fall in this rate of interest on the price of (a) War Loan, (b) land, (c) houses, (d) vintage port?
  6. Under what conditions will a check to the supply of a factor of production cause a large increase in its price?

B.

  1. Give a brief account of the modern English banking system, explaining the conditions under which the joint-stock banks can increase their funds in emergency.
  2. What do you understand by the purchasing power of money?
  3. What is meant by the balance of trade? Under what conditions is it possible for a country’s trade returns to show a continuous excess of exports over imports?
  4. What is the difference between a gold standard and a gold-exchange standard? Give examples.
  5. What determines the rate of exchange between the pound sterling and the American dollar?
  6. Is it possible for a country to be undersold all round by its competitors?

 

ESSAY.
(NEW REGULATIONS.)

TUESDAY, JUNE 2, 1931. 1.30—4.30.

Write an essay on one of the following subjects:

  1. “It is a kind of Proverb attending the Character of English Men, that they are better to improve than to invent” (Defoe).
  2. Soviet Farming.
  3. “That outpost of economic empire—the Argentine Republic.”
  4. The United States of Europe.
  5. “Ill fares the land, to hastening ills a prey, where wealth accumulates and men decay.”
  6. England’s Decline and Fall.

 

ENGLISH ECONOMIC HISTORY.
(NEW REGULATIONS.)

WEDNESDAY, JUNE 3, 1931. 9—12.

  1. Give some examples of the way in which economic events have left their impress on economic doctrine.
  2. Contrast the fiscal policy of Gladstone with that of Joseph Chamberlain.
  3. Examine the reasons which led to the localization of the major portion of the woolen industry in the West Riding of Yorkshire.
  4. Estimate the importance of the Chartered Companies for the expansion of England’s foreign trade after 1600.
  5. Illustrate the attitude of Parliament during the nineteenth century towards
    (a) Monopolies, (b) Joint-stock enterprise.
  6. Estimate the services rendered to England by Lord Shaftesbury.
  7. “The industrial revolution was the precursor of a commercial revolution, which was just as important.” Comment.
  8. Illustrate from English history the qualities required from a great inventor.
  9. “War always brings prosperity to agriculture, and Peace, when it comes, depression.” Comment.
  10. Account for the rise of the domestic system in England.
  11. “The repeal of the Corn Laws was the most important political event between the first and second Reform Bills.” Discuss.

 

SOCIAL PROBLEMS.
(NEW REGULATIONS.)

WEDNESDAY, JUNE 3, 1931. 1.30—4.30.

  1. Explain the difficulties involved in any attempt to measure exactly the National Dividend of Great Britain, and to compare it at two dates.
  2. Describe briefly the results of any one investigation into the extent and causes of poverty.
  3. Give some account of the changes in the level of real wages in this country since 1800, with particular attention to changes since 1914.
  4. Consider the importance of the following as causes of unemployment: (1) labour turnover, (2) the need for a reserve of labour, (3) immobility of labour. In what ways can their effects be diminished?
  5. Compare the efficacy of different methods of wage payment as incentives to increase output.
  6. In what conditions do your think it desirable that a government should intervene to fix minimum wages in a trade? What powers of intervention does the Minister of Labour at present possess?
  7. Examine briefly the following methods of dealing with the present unemployment problem: (1) public works, (2) emigration, (3) leaving it to private enterprise.
  8. What means have been suggested for making industry more democratic? How far have they been successful?
  9. Do you consider that the real incomes of workers can be increased more effectively by a rise of wages or by an increased expenditure upon social services?
  10. “The strength of Trade Unionism has been the greatest obstacle to improvements of industrial methods.”
    “The most powerful incentive to improvement has been the encroachment of wages upon the profits of employers.”
    Where does the truth lie?
  11. Give some account of the system of Unemployment Insurance in this country. Subject to what conditions may a man or woman draw benefit? What changes would you suggest in the present regulations?

 

PART II.

ECONOMIC PRINCIPLES.
(OLD AND NEW REGULATIONS.)

MONDAY, JUNE 1, 1931. 9—12.

  1. How far do you consider the theory of Economics still to depend upon the hypothesis of an economic man?
  2. “The theory of rent is nowadays not even of academic interest.” Discuss.
  3. “The mechanism of increasing returns is not to be discerned adequately by observing the effects of variations in the size of an individual firm or of a particular industry, for the progressive division and specialization of industries is an essential part of the process by which increasing returns are realized.” Discuss the truth and significance of this conclusion of Professor Allyn Young.
  4. If an employer finds it necessary to reduce output temporarily, what are the factors he should take into account when deciding whether
    1. to dismiss some of his employees,
    2. to maintain the working-force on short time?
  5. “The doctrine that the earnings of a worker tend to be equal to the net product of his work has by itself no real meaning; since in order to estimate net product we have to take for granted all the expenses of production of the commodity on which he works, other than his own wages.” Comment.
  6. Examine the argument that reductions in wage-rates, since they diminish the purchasing power of important bodies of consumers, can only aggravate a trade depression.
  7. “A wise national policy would seek to maintain an economic return on the vast capital invested in the English railways by attracting back to them a large part of the traffic which in recent years has been diverted to the roads.” Discuss this contention, making plain the economic principles involved.
  8. Examine the arguments for a monetary policy which permits the commodity price-level to fall in proportion to increases in general productivity.
  9. In a world in which no net annual addition is being made to the stock of capital, would you expect the rate of interest to fall to zero?
  10. “The emphasis laid in modern economic treatises on the theory of exchange value is misplaced. The true subject of economics is not the terms on which goods exchange for one another, but the forces which determine the magnitude of a country’s productive resources and the uses to which they are put.” Discuss.
  11. How much truth, if any, is there in the statement that “exports pay for imports”?

 

STRUCTURE AND PROBLEMS OF INDUSTRY.
(OLD REGULATIONS.)

MONDAY, JUNE 1, 1931. 1½ —4½.

  1. What are the main changes in the localisation of manufacturing industry in Great Britain since the War? To what influences do you attribute them?
  2. What difficulties would you meet in attempting to compare costs of production in the same industry in different countries?
  3. In the English cotton industry the typical firm is specialised either to spinning or to weaving; in other countries the typical firm combines spinning and weaving. How do you account for this difference? and do you expect it to persist?
  4. “Rationalisation is merely a new name for monopoly.” “Rationalisation is merely a euphemism for company-promoting.” Criticise these statements, and explain, with reference to some one industry, what you understand by Rationalisation.
  5. British exports to the East of tobacco, oil and fertilisers are distributed by subsidiaries of the producing companies; most other exports are sold by export merchants to merchants at the ports, who in turn leave the internal distribution to native dealers. What are the reasons for this difference of practice? Could the former method be applied with advantage to other exports?
  6. If you were engaged by an American investment trust to advise them on the investment of a portion of their funds in ordinary shares in this country, in what industries, and for what reasons, would you recommend investment?
  7. “Wage-rates in 1929 were approximately at the same level as in 1924, while the cost of living had fallen 5 per cent.; real wages had, therefore, risen about 5 per cent. But the Board of Trade index of industrial production showed an increase in 1929 of 11 per cent. over 1924; therefore there was a case for raising money wages.” Examine this argument.
  8. Explain the operation of either the Federal Farm Board or the Canadian Wheat Pool or the San Paulo Coffee Institute or the British Australian Wool Realisation Association. To what do you attribute its failure or success?
  9. Compare broadly the English and the German attitudes to Restraint of Trade… Explain the functions of the German Kartels Court, and discuss the suitability of such a Court to English conditions.
  10. In what sense, if at all, is there a Science of Management? Can Industrial Administration be taught?
  11. In England Local Authorities are obliged to arrange for the amortization within a definite period of all loans raised for the purpose of financing productive undertakings. Do you regard this as an undesirable handicap on public as compared with private enterprise?

 

STRUCTURE AND METHODS OF GOVERNMENT IN THE MODERN WORLD.
(OLD REGULATIONS.)

MONDAY, JUNE 1, 1931. 1½ —4½.

  1. Describe some of the methods by which central control of local government is provided for in modern States, and discuss their success.
  2. Discuss the view that the Referendum and the Initiative, whatever may be their merits under other systems, are incompatible with Cabinet government.
  3. Compare the means for securing that effective government shall be carried on in an emergency in the United Kingdom and the German Reich respectively.
  4. Compare the system of relationships between politicians and civil servants in England with that in France or any other country.
  5. Explain the unique strength of the United States Senate among contemporary Second Chambers, and discuss the part it plays in American government.
  6. Discuss whether the German Reich should more correctly be described as a federal or a unitary State.
  7. Compare the parts played by parties in the systems of government of the United Kingdom and the United States respectively.
  8. How far may Great Britain be said to fall behind continental countries in providing the individual with legal remedies against State action?
  9. “The constitutional position of the President in the post-war European republics follows the French rather than the American model.” How far is this true, and how do you account for what has occurred?
  10. Illustrate by reference to the respective constitutional positions of the Governor of a Crown Colony, the Viceroy of India, the Governor of an Indian Province, the Governor of an Australian State, and the Governor-General of a Dominion, the development in the office of the “royal governor.”
  11. Illustrate from the constitutions of the British Empire possible methods of safe-guarding the interests of racial or religious minorities (or majorities) where the population is not homogeneous.

 

MONEY, CREDIT AND PRICES.
(OLD REGULATIONS.)

TUESDAY, JUNE 2, 1931. 9—12.

  1. “I am never wary of preaching in the wilderness ‘the only very important thing to be said about currency is that it is not nearly as important as it looks.’” (Marshall.) Do you agree?
  2. Indicate the circumstances in which a Central Bank can most effectively enforce its policy by (a) changes in the bank-rate, (b) open-market operations, (c) rationing of credit.
  3. Has the recent growth of the hire-purchase system for consumption-goods either precipitated or aggravated the slump in the U.S.A.?
  4. “It is even conceivable that the cash-deposits may remain the same, the savings-deposits may remains the same, the volume of monetary transactions may remain the same, and the volume of output may remain the same; and yet the fundamental price-levels may change.” (Keynes.) Is this conceivable? And if so, does it mean that the traditional version of the quantity theory of money is not merely a truism but an error?
  5. In what circumstances would the successful stabilization of the purchasing power of money in terms of an index-number of prices tend to increase or to diminish the fluctuations in the prices of individual commodities?
  6. Factors affecting the general price-level have been distinguished as acting “on the side of money” and “on the side of goods.” Criticize this distinction, or define it so as to avoid ambiguities.
  7. “The belief that the elasticity of demand for currency can ever be different from unity is based on a misunderstanding of what elasticity of demand is.” Discuss.
  8. During 1930 the imports of gold into France have exceeded the world output of gold. What were the causes of this influx?
  9. If central banks fixed their buying and selling prices of gold wider apart, would the diffusion of trade fluctuations from country to country be checked?
  10. How would the internal price-level of this country be affected by a special tax on incomes from foreign investments?
  11. “The joint-stock banks have attracted to their custody a larger volume of the country’s savings than can be economically employed in short-term commercial credits. Some modification in the traditional practice of English deposit banking is therefore required.” Discuss.

 

INTERNATIONAL LAW.
(OLD REGULATIONS.)

TUESDAY, JUNE 2, 1931. 9—12.

  1. Estimate the present international status (if any) of
    1. Canada;
    2. Bavaria;
    3. the State of Virginia;
    4. the Vatican City;
    5. the Territory of the Saar Basin.
  2. Describe the present state of the movement for the codification of International Law (a) as to Peace, (b) as to War and Neutrality. What are in your opinion the chief obstacles in each case?
  3. You are private secretary to a member of Parliament who sits for a fishing constituency bordering on the Moray Firth and who is requested by his constituents to address them upon what they describe as “the invasion of our waters” by Dutch fishing trawlers. He asks you to coach him upon the law (apart from any statutes) relating to fishing by foreign trawlers (a) in territorial waters and (b) in bays, and also (c) upon a proposal to fix by international convention the breadth of all territorial waters at twelve miles. Advise him.
  4. Explain the expression occurring in Article 15 of the Covenant of the League: “a matter which by International Law is solely within the domestic jurisdiction” of one party to the dispute. Give some illustrations of such matters and refer to any judicial discussion of the expression. Is the category of such matters a fixed one?
  5. Discuss the assertion that “the mandate system adopted at the end of the Great War differs in name only from the old-fashioned system of annexation by the victor of the colonies of his defeated enemy.”
  6. Explain the operation of, and the difference between, “national treatment” and “most-favoured nation treatment” stipulated for in commercial treaties. What are the two principal interpretations of the usual “most-favoured-nation treatment” clause?
  7. Discuss the rules of International Law which are relevant to the growth of trading by Governments, including the rules governing the status of State-owned merchant ships in foreign ports.
  8. Describe, with illustrations, the different ways in which the Permanent Court of International Justice may acquire jurisdiction in respect of a dispute. What do you understand by “non-justiciable disputes”?
  9. Discuss the extent to which the conception of Contraband has changed since the beginning of the Great War, and the effect of that change upon the Declaration of Paris of 1856.
  10. A Government White paper published in 1929 concludes an argument with the sentence: “In other words, as between Members of the League, there can be no neutral rights because there can be no neutrals.” Discuss the accuracy of this statement, having regard both to the Covenant and to the Peace Pact of Paris.

 

SUBJECTS FOR AN ESSAY.
(OLD AND NEW REGULATIONS.)

TUESDAY, JUNE 2, 1931. 1½ —4½.

  1. Business Forecasting.
  2. The Decline and Fall of the British Empire.
  3. The Rights of Shareholders.
  4. “Buy British.”
  5. Republicanism.
  6. Arnold Bennett, the interpreter of industrialism.

 

POLITICAL THEORY.
(OLD REGULATIONS.)

WEDNESDAY, JUNE 3, 1931. 9—12.

  1. In what sense, and to what extent, do you regard it as the place of the State to allot their provinces to other associations?
  2. In what circumstances, if any, would you justify the refusal by the individual of a demand on the part of the State for his military service, and on what grounds?
  3. “La liberté est le droit de faire tout ce que les lois permettent.” Montesquieu.
    “By liberty I mean the assurance that every man shall be protected in doing what he believes his duty against the influence of authority and majorities, custom and opinion.” Acton.
    Discuss the adequacy of these definitions.
  4. Wherein do you consider the originality and importance of Rousseau as a political theorist to lie?
  5. Compare and contrast Mazzini’s doctrine of nationality with those of present-day Italian nationalists.
  6. “The citizen should be moulded to suit the form of government under which he lives.” To what extent do you accept this principle of Aristotle’s?
  7. Examine the implications and discuss the validity of the theory of a “right to work or maintenance.”
  8. What circumstances are required, in your opinion, to justify a demand for “national self-determination” on the part of a community?
  9. Consider the arguments for increased public control of the Press in England.
  10. How far, in your opinion, does (a) illiteracy, (b) failure to vote, (c) ecclesiastical influence justify the withholding or withdrawal of the political franchise?

 

DISTRIBUTION AND LABOUR.
(OLD REGULATIONS.)

WEDNESDAY, JUNE 3, 1931. 9—12.

  1. Examine critically the policy of reducing unemployment by settling new industries in districts where older industries appear to be permanently depressed.
  2. Contrast conventional and “scientific” methods of determining “fair” piece-rates. What are the reasons for varying the rate degressively or progressively according to the total output produced?
  3. Discuss the difficulties of demonstrating statistically that wages tend to equality in trades “which are of equal difficulty and disagreeableness, which require equal natural abilities and an equally expensive training.”
  4. “In the past he had often had occasion to speak of ‘insurance popularly miscalled the dole.’ To-day he was afraid that it might be truer to speak of ‘the dole officially miscalled insurance.’” Discuss this view of Sir William Beveridge expressed in his evidence before the Royal Commission on Unemployment Insurance.
  5. Show by reference to specific trades the reasons for which (a) Trade Boards, (b) Joint Industrial (“Whitley”) Councils have been established in some trades and not in others. Can you suggest possible extensions of either to any further trades?
  6. What effect would (a) the growth of trade unionism among women, (b) the repeal of legal restrictions upon women’s employment, be likely to have upon the wages and employment of men?
  7. What measures may any one employer take to reduce labour costs besides reducing wage-rates or substituting machines for men? Is there any statistical evidence of the exact effect of such measures?
  8. What considerations would you take into account in adjudicating on a claim for reduction of wages in an industry in which no return is being earned on ordinary capital and there is over 15 per cent. unemployment?
  9. How far do you consider the marginal theory of distribution a full and adequate explanation of the actual distribution of incomes among persons?
  10. Analyse the problem of regulating wages and hours in the coal-mining industry as illustrated by events since the war.

 

PUBLIC FINANCE.
(OLD AND NEW REGULATIONS.)

WEDNESDAY, JUNE 3, 1931. 1½ —4½.

  1. Examine the view that the budgetary problems of the creditor countries concerned would be lightened rather than aggravated by a general remission of reparations and war-debts.
  2. The Royal Commission on Transport, reporting at the end of 1930, proposed that the present proportion in which the cost of the roads is shared between Local Authorities and the users of mechanical transport should be reversed, the share of the latter being brought up to two-thirds by the payment into the Road fund of the then existing duty on petrol and of that part of the licence duties on motor vehicles which is at present diverted to the Exchequer. Discuss this proposal.
  3. If you were in charge of the finances of the U.S.S.R., on what part, if any, of the capital employed in the various nationalized industries would you require the payment of interest?
  4. Is there any ground for the opinion that taxation imposed for the purpose of paying interest on internal debt is less onerous to the nation than taxation imposed for the purpose of paying the salaries of Government officials?
  5. Examine the view that improvements in the means of transport, such as suburban Tube extensions, should be financed out of confiscatory taxes on the resultant increments in the site value of surrounding land.
  6. If, as Chancellor of the Exchequer, you had £10 millions a year to dispose of, what considerations would you take into account in deciding between the claims for a further subsidy to working-class housing and those of a subsidy to domestic wheat-growing?
  7. “The arguments again the taxation of imported food which were valid in the early years of the twentieth century have one and all ceased to be valid today.” Comment.
  8. “As soon as the services financed by Government come to exceed the minimum requirements of security and order, the whole notion of equity in taxation becomes unreal and meaningless.” Discuss.
  9. How should the maintenance of those unemployed persons who cannot be brought within the framework of a self-supporting insurance scheme be financed?
  10. “The annual repayment of a substantial block of the National Debt is a fetish to which successive Chancellors of the Exchequer pay lip service, but which no sensible person either expects or desires to see carried out.” Comment.
  11. Discuss broadly the special problems of Public Finance which arise in countries with a Federal form of Government, illustrating your answer by reference to any one of the following countries: The United States, Germany, Australia, India.

 

STATISTICS.
(OLD AND NEW REGULATIONS.)

THURSDAY, JUNE 4, 1931. 9—12.

  1. Define the terms median and quartiles, and discuss the uses, advantages and disadvantages of the median as an average and of the semi-interquartile range as a measure of dispersion.
    Find the medians and quartiles of the two series of index-numbers for prices of twenty foodstuffs below, and use the work to illustrate your comments.
1913 1929
1. 58 77
2. 65 91
3. 66 84
4. 70 91
5. 73 95
6. 73 112
7. 67 95
8. 82 143
9. 92 120
10. 98 132
11. 99 142
12. 102 151
13. 105 175
14. 104 157
15. 95 144
16. 40 42
17. 38 31
18. 88 140
19. 44 54
20. 52 93
  1. Sketch the forms of frequency distributions most commonly met with in practice, and name sources from which examples of each can be drawn.
    Find the mean and standare deviation of the distribution below. Earnings were given to a penny.
Earnings Hands with earnings between limits stated
20s. but less than 25s. 6
25s. but less than 30s. 107
30s. but less than 35s. 490
35s. but less than 40s. 228
40s. but less than 45s. 109
45s. but less than 50s. 33
50s. but less than 55s. 14
55s. but less than 60s. 6
60s. but less than 65s. 4
65s. but less than 70s. 2
70s. but less than 75s. 1
Total 1000

 

  1. State the formula for the correlation coefficient and deduce its principal properties.
    The correlation coefficient between the two series of index-numbers given in Question 1 was calculated, and the following figures obtained:
1913 1929
Arbitrary origins used 76 108
Sums of deviations from these origins -9 +9
Sums of squares of deviations from these origins 9131 30,055
Sum of products of deviations +15, 834

Find the coefficient, and the regression equations.

  1. Give any deduction you prefer of the normal curve of errors, and prove its principal properties. Describe the tables of functions of the normal curve that are available in such a volume as Tables for Statisticians and Biometricians, and their uses in solving problems in sampling.
  2. Show that if random samples of n observations are drawn from an indefinitely large record in which the proportion of A’s is p and of not-A’s is q, the standard deviation of the numbers of A’s in the samples is .
    You are given the following data:
    Father skilled worker:—36 boys: 24 classed as intelligent.
    Father unskilled: —64 boys: 32 classed as intelligent.
    Would you say that the difference was significant?
  3. Give a description and critical discussion of the methods available for investigating the relations between two quantities varying with the time.
  4. Argue the case for the use of the geometric mean in connection with index-numbers of wholesale prices, comparing the advantages and disadvantages against other methods. Name any existing index-number for which it is used, and describe the detailed method of construction.
  5. Indicate the difficulties that are met with in obtaining from Census and Registration data reasonably accurate measures of the mortalities of men engaged in different occupations, and describe the technical methods that are used for indicating relative mortalities.
  6. The following are the ages returned at the Census of 1921 by a sample of male Hindus in Madras. By some process of graduation, make an estimate of the true numbers at 30 and 31 years of age.
Age Numbers Age Numbers Age Numbers
20 8579 30 12294 40 10391
21 1077 31 652 41 460
22 3053 32 2058 42 1105
23 1156 33 672 43 436
24 1786 34 892 44 514
25 9291 35 7723 45 5352
26 1946 36 1437 46 957
27 1595 37 870 47 570
28 2709 38 1362 48 919
29 927 39 467 49 395

INDUSTRY.
(NEW REGULATIONS.)

MONDAY, JUNE 1, 1931. 1½ —4½.

  1. What difficulties would you meet in attempting to compare costs of production in the same industry in different countries?
  2. In the English cotton industry the typical firm is specialised either to spinning or to weaving; in other countries the typical firm combines spinning and weaving. How do you account for this difference? and do you expect it to persist?
  3. British exports to the East of tobacco, oil and fertilisers are distributed by subsidiaries of the producing companies; most other exports are sold by export merchants to merchants at the ports, who in turn leave the internal distribution to native dealers. What are the reasons for this difference of practice? Could the former method be applied with advantage to other exports?
  4. “Wage-rates in 1929 were approximately at the same level as in 1924, while the cost of living had fallen 5 per cent.; real wages had, therefore, risen about 5 per cent. But the Board of Trade index of industrial production showed an increase in 1929 of 11 per cent. over 1924; therefore there was a case for raising money wages.” Examine this argument.
  5. Explain the operation of either the Federal Farm Board or the Canadian Wheat Pool or the San Paulo Coffee Institute or the British Australian Wool Realisation Association. To what do you attribute its failure or success?
  6. Compare broadly the English and the German attitudes to Restraint of Trade. Explain the functions of the German Kartels Court, and discuss the suitability of such a Court to English conditions.
  7. In what sense, if at all, is there a Science of Management? Can Industrial Administration be taught?
  8. “The only trades which it seems possible for a joint-stock company to carry on successfully, without an exclusive privilege, are those, of which all the operations are capable of being reduced to what is called a routine, or to such a uniformity of method as admits of little or no variation” (Adam Smith).
    Examine this statement in the light of the subsequent development of the joint-stock company.
  9. What features has the present depression in British industry in common with those of the nineteenth century?
  10. In what directions, and with what results, did England export capital in the half-century before the War?
  11. In England Local Authorities are obliged to arrange for the amortization within a definite period of all loans raised for the purpose of financing productive undertakings. Do you regard this as an undesirable handicap on public as compared with private enterprise?

 

MONEY.
(NEW REGULATIONS.)

TUESDAY, JUNE 2, 1931. 9—12.

  1. “I am never wary of preaching in the wilderness ‘the only very important thing to be said about currency is that it is not nearly as important as it looks.’” (Marshall.) Do you agree?
  2. Indicate the circumstances in which a Central Bank can most effectively enforce its policy by (a) changes in the bank-rate, (b) open-market operations, (c) rationing of credit.
  3. “It is even conceivable that the cash-deposits may remain the same, the savings-deposits may remains the same, the volume of monetary transactions may remain the same, and the volume of output may remain the same; and yet the fundamental price-levels may change.” (Keynes.) Is this conceivable? And if so, does it mean that the traditional version of the quantity theory of money is not merely a truism but an error?
  4. Factors affecting the general price-level have been distinguished as acting “on the side of money” and “on the side of goods.” Criticize this distinction, or define it so as to avoid ambiguities.
  5. “The adoption of Ricardo’s currency plan in 1925, as well as its rejection a hundred years before, point to the same conclusion: the economic ideas of legislators are always a century out of date.” Comment.
  6. Describe the main fluctuations in the value of silver during the last sixty years, and state their causes.
  7. What are the chief changes in the organization of the London money market since the publication of Bagehot’s Lombard Street?
  8. During 1930 the imports of gold into France have exceeded the world output of gold. What were the causes of this influx?
  9. If central banks fixed their buying and selling prices of gold wider apart, would the diffusion of trade fluctuations from country to country be checked?
  10. How would the internal price-level of this country be affected by a special tax on incomes from foreign investments?
  11. “The joint-stock banks have attracted to their custody a larger volume of the country’s savings than can be economically employed in short-term commercial credits. Some modification in the traditional practice of English deposit banking is therefore required.” Discuss.

 

LABOUR.
(NEW REGULATIONS.)

WEDNESDAY, JUNE 3, 1931. 9—12.

  1. Examine critically the policy of reducing unemployment by settling new industries in districts where older industries appear to be permanently depressed.
  2. Contrast conventional and “scientific” methods of determining “fair” piece-rates. What are the reasons for varying the rate degressively or progressively according to the total output produced?
  3. What effect would (a) the growth of trade unionism among women, (b) the repeal of legal restrictions upon women’s employment, be likely to have upon the wages and employment of men?
  4. What measures may any one employer take to reduce labour costs besides reducing wage-rates or substituting machines for men? Is there any statistical evidence of the exact effect of such measures?
  5. What considerations would you take into account in adjudicating on a claim for reduction of wages in an industry in which no return is being earned on ordinary capital and there is over 15 per cent. unemployment?
  6. How far do you consider the marginal theory of distribution a full and adequate explanation of the actual distribution of incomes among persons?
  7. What is the statistical evidence as to changes in the real earnings of the wage-earning classes and their living conditions during the last hundred years? How far do you think this evidence conclusive?
  8. Compare the post-war attitude of governments and governing classes to the relief of poverty with the attitude prevailing in the Victorian era.
  9. Analyse the problem of regulating wages and hours in the coal-mining industry as illustrated by events since the war.
  10. Account for the fluctuations in the emphasis placed by Trade Unions upon the policies of mutual insurance, political action and the strike, respectively, during the last hundred years.

 

Source: Cambridge University. Economics Tripos Papers 1931-1933. Cambridge: 1933, pp. 5-27.

Image Source: King’s College dining hall, Cambridge, from Wikipedia Commons.

Categories
Bibliography Exam Questions Harvard Suggested Reading Syllabus

Harvard. Money And Banking. Readings and Exams. Williams and Hansen, 1947-48

 

The graduate course for Keynesian economics at Harvard in the 1940s was Principles of Money and Banking taught by Alvin H. Hansen and John H. Williams. Course materials for 1946-47 were transcribed and posted earlier [Fall term 1946; Spring term 1947; General course bibliography]. Almost all of the exam questions for 1947-48 are new. The Spring term of 1948 taught by John  Williams turns out to be unchanged from the previous year. The Fall term of 1947 taught by Alvin Hansen does show some minor rearrangements, and significant additions (e.g. Tobin on liquidity preference).

____________________________

Course Enrollment
1947-48

[Economics] 141a. Professors Williams and Hansen. — Principles of Money and Banking (F).

Total 81: 47 Graduates, 1 Senior, 20 Public Administration, 4 Business, 9 Radcliffe.

 

[Economics] 141b. Professors Williams and Hansen. — Principles of Money and Banking (Sp).

Total 70: 41 Graduates, 2 Juniors, 20 Public Administration, 2 Business, 5 Radcliffe.

 

Source: Harvard University. Report of the President of Harvard College and Reports of Departments for 1947-48, p. 91.

 

____________________________

ECONOMICS 141
PRINCIPLES OF MONEY AND BANKING

 

Economics 141a — First Semester, 1947-8 (Professor Hansen)

  1. Central Banking: Current Problems and Policies
  2. Theory of Money, Liquidity-Preference, Interest and Prices

 

Economics 141b — Second Semester, 1947-8 (Professor Williams)

  1. International Monetary Equilibrium
  2. Monetary and Fiscal Policy

 

READING LIST FOR ECONOMICS 141a
Principles of Money and Banking
1947-1948

 

Note: Pre-requisite reading (for those who are deficient in undergraduate preparation in Money and Banking:

  1. Banking Studies, Board of Governors, Federal Reserve System, (1941).
  2. Southard, F. A., Foreign Exchange Practice and Policy, (McGraw-Hill, 1940).
  3. Any one standard textbook in Money and Banking, such as: Thomas, Our Modern Banking and Monetary System, (Prentice-Hall, 1942); or Reed, Money, Currency and Banking, (McGraw-Hill, 1942).

 

  1. Central Banking: Current Problems and Policies.
    1. Minimum Reading List:
      1. Books and Pamphlets:
        1. International Currency Experience (League of Nations, 1944), Chapters I-IV, pp. 7-112.
        2. World Economic Survey, 1942-44 (League of Nations, 1945), Chapter IV “Finance and Banking” (pp. 173-213).
        3. Ellis, H. S., (in Harris: Economic Reconstruction, McGraw-Hill, 1945), Chapter 13, “Central and Commercial Banking in Postwar Finance” (pp. 237-252).
        4. Hansen, Alvin H., America’s Role in the World Economy (Norton, 1945), Chapter XVII, “Gold, Exports and Liquidity” (pp. 144-157).
        5. Harris, S. E., Inflation and the American Economy (McGraw-Hill, 1945), Chapter XXIV, “Money and Savings” (pp. 372-383).
        6. Hawtrey, R. G., The Art of Central Banking (Longmans, 1933) pp. 116-207.
        7. Keynes, J. M., Treatise on Money, Volume II, Chapters 25, 32, 33, (pp. 49-78; 225-278).
        8. Robertson, D. H., Essays in Monetary Theory (King, 1940), Chapter II, “Theories of Banking Policy” (pp. 39-59); Chapter XII, “British Monetary Policy” (pp. 154-167).
        9. Williams, John H., Postwar Monetary Plans (Knopf, second edition, 1945), Chapter 6, “The Banking Act of 1935” (pp. 112-129); Chapter 8, “The Crisis of the Gold Standard” (pp. 154-172); Chapter 9, “Monetary Stability and the Gold Standard” (pp. 172-190).
        10. Financing American Prosperity (Twentieth Century Fund, 1945):
          1. Ellis, H. S., “Monetary Controls and the Business of Banking” (pp. 140-153).
          2. Williams, John H., “Money and Banking” (pp. 381-5).
        11. Postwar Economic Studies, No. 3 (Board of Governors, Federal Reserve System, 1945): Wallich, H. C., “Public Debt and Income Flow” (pp. 84-100).
        12. Hansen, Alvin H., Economic Policy and Full Employment, Chapters 20 and 22 (pp. 233-247; 261-288).
      2. Reports and Articles:
        1. Treasury Bulletin, April, 1946, “Federal War-time Financing and Growth of Liquid Assets”, pp. A11-20.
        2. Federal Reserve Bulletins:
          1. July, 1947, “Debt Retirement” (pp. 775-87); “Consumer Incomes and Liquid Assets” (pp. 788-802); “International Monetary and Financial Problems” (pp. 836-850).
          2. April, 1947, “Economic Survey of the United Kingdom” (pp. 367-391); “Annual Report of the Bank of Canada” (pp. 392-97); “Monetization of Public Debt by Banks” (pp. 402-04).
          3. “Estimated Liquid Assets of Individuals and Business”, November, 1946, pp. 1236-37; June, 1947, pp. 689-91.
        3. Annual Reports of Board of Governors, Federal Reserve System:
          1. Thirty-second Report (for the year 1945) pp. 1-15.
          2. Thirty-third Report (for the year 1946) pp. 1-49.
        4. Bopp, K. R., “Central Banking at the Crossroads”, Supplement, American Economic Review, March 1944 (pp. 260-77).
        5. Samuelson, Paul, “The Effect of Interest Rate Increases on the Banking System”, American Economic Review, March 1945.
        6. Seligman, H. L., “The Problem of Excessive Commercial Bank Earnings”, Quarterly Journal of Economics, May 1946.
        7. Whittlesey, C. R., “Federal Reserve Policy in Transition”, Quarterly Journal of Economics, May 1946.
    2. Supplementary Reading List:
      1. Books
        1. Arndt, H. W., The Economic Lessons of the Nineteen Thirties, (Oxford, 1944).
        2. Coulborn, W, A. L., An Introduction to Money, (Longmans, 1938) Chapters 5, 13-14 (pp. 48-64, 209-241).
        3. Fisher, Irving, 100 Per Cent Money, (Adelphi, 1935; Third Edition City Printing Co., New Haven, 1945).
        4. Johnson, G. G., The Treasury and Monetary Policy, (Harvard 1939), Chapter I-V (pp. 3-160)
        5. Hawtrey, R. G., The Gold Standard in Theory and Practice (Longmans, Fourth Edition, 1939).
        6. Hawtrey, R. G., A Century of Bank Rate. (Longmans, 1938).
        7. Lewinski, J., Money, Credit and Prices, (King, 1929) Chapters IV-V (pp. 99-144).
        8. McCracken, Paul W., The Future of Northwest Bank Deposits, Federal Reserve Bank, Minneapolis, 1946.
        9. Mints, L. W., A History of Banking Theory (Chicago, 1945), Chapters VI and X (pp. 74-100; 178-197).
        10. Morgan, E. V., The Theory and Practice of Central Banking, (Macmillan, 1943).
        11. Niebyl, Karl H., Studies in the Classical Theories of Money, (Columbia, 1946).
        12. Sayers, R. S., Modern Banking, (Oxford, 1938), Chapters 4-5 (pp. 70-145).
        13. Viner, J. Studies in the Theory of International Trade, (Harper, 1937), Chapter V, “English Currency Controversies” (pp. 218-289).
        14. Wernette, P., Financing Full Employment, (Harvard, 1945), Chapter 3 (pp. 33-61).
        15. Macmillan Report, Royal Commission in Industry and Commerce, Cmd. 3897 (1931) pp. 2-45; 106-160.
      2. Articles
        1. Abbott, C. C. (Review articles on Financing Problems and Bank Liquidity), Review of Economic Statistics, February 1946 (pp. 48-51).
        2. Abbott, C. C., “Management of the Federal Debt”, Harvard Business Review, Autumn 1945.
        3. Goldenweiser, E. A., “Commercial Banking After the War”, Federal Reserve Bulletin, September 1944.
        4. Seltzer, Lawrence, “Is a Rise in Interest Rates Desirable or Inevitable?”, American Economic Review, December 1945.
        5. Treasury Bulletin, April 1946, “Federal War-time Financing and the Growth of Liquid Assets”.
        6. Keynes, J. M., “The Objective of International Price Stability”, Economic Journal, June-September 1943.
    3. General Reference Reading (see below).

 

  1. Theory of Money, Liquidity Preference, Interest and Prices.
    1. Minimum Reading List:
      1. Books:
        1. Fellner, William, Monetary Policies and Full Employment, Chapter 6, (pp. 174-209).
        2. Hansen, Alvin H.:
          1. Economic Policy and Full Employment, Chapters 12, 13, 18, 19 and 21, (pp. 145-160; 202-232; 248-260).
          2. Fiscal Policy and Business Cycles, (Norton, 1941), Chapters 1-5; 11-15; (pp. 13-105; 225-338).
          3. Full Recovery or Stagnation, (Norton, 1938), Chapter 3 (pp. 59-87); Appendix, pp. 331-343.
        3. Hayek, F. A., Prices and Production, (Routledge, 1935), Chapters 1 and 4 (pp. 1-31; 105-128).
        4. Keynes, J. M., Monetary Reform, (Harcourt, 1924), pp. 81-95; 152-191.
        5. Keynes, J. M., A Treatise on Money, (Harcourt, 1930), Chapters 9-13 and 30 (Volume I, pp. 123-220; Volume II, pp. 148-208).
        6. Keynes, J. M., General Theory of Employment, Interest and Money, (Harcourt, 1936), pp. 3-45; 61-65; 74-221; 245-271; 292-332; 372-384.
        7. Klein, Lawrence, The Keynesian Revolution, Chapters 1-3, (pp. 1-90).
        8. Marget, Arthur W., The Theory of Prices, Volume I, (Prentice-Hall, 1938), Chapters 12 and 15 (pp. 302-343, 414-459, and large type sections).
        9. Marget, Arthur W., The Theory of Prices, Volume II, (Prentice-Hall, 1942), Chapter 3 (pp. 89-133, large type sections).
        10. Marshall, A., Money, Credit and Commerce, (Book I, Chapter XX, pp. 38-50.
        11. Robertson, D. H., Essays in Monetary Theory, (King, 1940), Chapters 1, 6, 11 (pp. 1-38; 92-7; 113-153).
        12. Schumpeter, J. A., Business Cycles, (McGraw-Hill, 1939), Volume II, Chapter 8, (pp. 449-482).
        13. Wicksell, K., Interest and Prices, (Macmillan, 1936), Introduction by Bertil Ohlin; also author’s Preface; Chapters 5, 7-8, 11 (pp. 38-50; 81-121; 165-177).
        14. Wicksell, K., Money: Lectures on Political Economy, Volume II, (Macmillan, 1935), Chapter IV (pp. 127-228).
        15. Wright, David McC., The Creation of Purchasing Power, (Harvard, 1939), Chapters 4-6 (pp. 60-121).
        16. Macmillan Report, Royal Commission on Finance and Industry, Cmd. 3897 (1931), Part I, Chapter 11 (pp. 92-105).
      2. Articles:
        1. Clark, Colin, “Public Finances and Changes in the Value of Money”, Economic Journal, December 1945.
        2. Hicks, J. R., “Mr. Keynes and the Classics: A Suggested Interpretation”, Econometrica, April 1937.
        3. Hawtrey, R. G. and Hicks, J. R., “Interest and Bank Rate”, The Manchester School of Economic and Social Studies, October 1939.
        4. Harrod, Hansen, Haberler, and Schumpeter, “Keynes’ Contribution to Economics”, Review of Economic Statistics, November, 1946.
        5. Keynes, J. M., “Relative Movement of Real Wages and Output”, Economic Journal, March 1939.
        6. Lange, O., “The Rate of Interest and the Optimum Propensity to Consume”, Economica, February 1938.
        7. Lerner, A. P., “Interest Theory: Supply and Demand for Loans or Supply and Demand for Cash”, Review of Economic Statistics, May 1944.
        8. Mints, Hansen, Ellis, Lerner, Kalecki, “A Symposium on Fiscal and Monetary Policy”, Review of Economic Statistics, May 1946.
        9. Modigliani, F., “Liquidity Preferences and the Theory of Interest and Money”, Econometrica, January 1944.
        10. Simons, H. C., “Debt Policy and Banking Policy”, Review of Economic Statistics, May 1946.
        11. Tobin, James, “Liquidity Preference and Monetary Policy”, The Review of Economic Statistics, May 1947.
    2. Supplementary Reading List:
      1. Books:
        1. Adarkar, B. P., The Theory of Monetary Policy, (King, 1935), Chapter 1-8; 13-15 (pp. 3-52; 101-122).
        2. Chandler, L. V., An Introduction to Monetary Theory (Harper, 1940), pp. 1-205.
        3. Coulborn, W. A. L., An Introduction to Money, (Longmans, 1938), Chapters 6-8; 15-16 (pp. 65-116; 242-264).
        4. Haberler, G., Prosperity and Depression (1939) Chapters 8, 13 (pp. 168-254; 455-507).
        5. Hicks, J. R., Value and Capital, Chapters 12-13.
        6. Lindahl, Erik, Studies in the Theory of Money and Capital, (Allen and Unwin, 1939), Part II, Chapters 4-6, (pp. 199-268).
        7. Myrdal, Gunnar, Monetary Equilibrium, (Hodge, 1939), Chapters 1-3 (pp. 1-48).
        8. Polanyi, M. Full Employment and Free Trade, (Cambridge Univ. Press, 1945), Chapters 1, 4, (pp. 1-66; 87-103).
        9. Robertson, D. H., Money (Harcourt, 1929) Chapters 2-4; 7-8.
        10. Sayers, R. S., Modern Banking. (Oxford, 1938), Chapter 6 (pp. 146-164).
        11. Thomas, Brindley, Monetary Policy and Crises, (Routledge, 1936), Chapters 3-4 (pp. 62-156).
      2. Articles:
        1. Lange, O., “Economic Controls After the War,” Political Science Quarterly, March 1945.
        2. Lerner, A. P., “Alternative Formulations of the Theory of Interest”, Economic Journal, June 1938.
        3. Lerner, A. P., “Ex Ante Analysis and Wage Theory”, Economica, November 1939.
        4. Lerner, A. P., “Some Swedish Stepping Stones in Economic Theory”, Canadian Journal of Economics and Political Science, November 1940.
        5. Marschak, J., “Wicksell’s Two Interest Rates”, Social Research, November 1941.
        6. Simons, H. C., “On Debt Policy”, Journal of Political Economy, June 1945.
        7. Warburton, Clark, “The Volume of Money and the Price Level Between the World Wars”, Journal of Political Economy, June 1945.
        8. a. Warburton, Clark, “The Monetary Theory of Deficit Financing”, Review of Economic Statistics, May 1945.
          b. Arndt, H. W., “The Monetary Theory of Deficit Financing; A Comment”, Review of Economic Statistics, May 1946.
        9. Bean and others, “Five Views on the Consumption Function”, Review of Economic Statistics, November, 1946.
    3. General Reference Reading (see below).

Source: Harvard University Archives. Syllabi, course outlines and reading lists in Economics, 1895-2003 (HUC 8522.2.1) Box 4, Folder “Economics, 1947-48 (2 of 2)”.

____________________________

Mid-year Exam

1947-48
HARVARD UNIVERSITY
ECONOMICS 141a

Part A. Write on one question only.

  1. Write an essay on Federal war-time financing including a discussion of:
    1. The role played by (a) the Federal Reserve Banks, (b) the commercial banks.
    2. The impact on (a) the money supply, (b) the liquid assets, (c) member bank reserves, (d) currency in circulation, (e) the rate of interest.
  2. Discuss major problems currently confronting the Federal Reserve System including an appraisal of various proposals to deal with these problems.

Part B. Write on any three questions.

  1. Write an essay (historical and analytical) on the relation of the money supply to the national income. In this connection discuss: (a) the Quantity Theory (b) the Marshallian “k” and (c) the Keynesian liquidity preference functions.
  2. Using the diagrams and analysis of Hicks and Keynes, discuss the role of (a) the schedule of the marginal efficiency of capital (b) the consumption function (c) the liquidity preference function and (d) the quantity of money, as determinants of the rate of interest and of income.
  3. State precisely the conditions (in particular including the relevant functions and their interest-elasticities) under which Monetary Policy alone, or Fiscal Policy alone (without either being supplemented by the other) may be (a) fully effective, (b) wholly ineffective, in raising income.
  4. Write an essay on the “theory of prices” including a discussion of money, income, wage and cost functions; in particular make use of the Keynesian analysis contained in the General Theory, Book V. (Money, Wages, and Prices.)
  5. Write an essay on any one of the following:
    1. International Currency Experience (League of Nations).
    2. Hawtrey, The Art of Central Banking.
    3. Keynes: Treatise on Money.
    4. Robertson: Essays on Monetary Theory.
    5. Williams, Postwar Monetary Plans.
    6. Klein, The Keynesian Revolution.
    7. Wicksell: Interest and Prices.

Note: You will be expected to write on 4 questions (one from part A and three from Part B.

Final. January, 1948.

 

Source: Harvard University Archives. Harvard University Final Examinations 1853-2001. Box 15. Papers Printed for Final Examinations: History, History of Religions…, Economics, … , Military Science, Naval Science, January, 1948.

____________________________

 SECOND SEMESTER
ECONOMICS 141b: PRINCIPLES OF MONEY AND BANKING

  1. International Monetary Equilibrium:
    1. Cassel, G., The Downfall of the Gold Standard (1936).
    2. Copland, Douglas, Australia in the World Crisis (1934).
    3. Ellis, H. S., Exchange Control in Central Europe (1941).
    4. Graham and Whittlesey, Golden Avalanche (1939).
    5. Hall, M. F., The Exchange Equalization Account (1935).
    6. Hahn, George, International Monetary Cooperation (1945).
    7. Hansen, Alvin, H., America’s Role in the World Economy (1945).
    8. Hardy, C. O., Is There Enough Gold (1936).
    9. Harris, S. E., Exchange Depreciation (1936).
    10. Harris, S.E., Economic Problems of Latin America (1944).
    11. Iverson, Carl, International Capital Movements (1936).
    12. Kindelberger, C. P., International Short-term Capital Movements (1937).
    13. League of Nations, Final Report on Gold (1932).
    14. League of Nations, Economic Fluctuations in the United States and the United Kingdom, 1918-22 (1942).
    15. Nurkse, R., International Currency Experience (1944).
    16. Warren and Pearson, (a) Gold and Prices (1935);
      (b) World Prices and the Building Industry (1937).
    17. Williams, John H., Postwar Monetary Plans (Second Edition, 1945)
  2. Monetary and Fiscal Policy:
    1. Beveridge, Sir William, Full Employment in a Free Society (1945).
    2. British White Paper on “Employment Policy” (1944).
    3. de Chazeau, Hart, and Others, Jobs and Markets (1946).
    4. Economics of Full Employment. Six Oxford Economists (1945).
    5. Fellner, W., Monetary Policies and Full Employment (1946).
    6. Financing American Prosperity, Twentieth Century Fund (1945).
    7. Groves, H. M., (a) Production, Jobs and Taxes (1944).
      (b) Postwar Taxation and Economic Progress (1946).
    8. Hansen, Alvin, H., Economic Policy and Full Employment (1946).
    9. Harris, S. E., Postwar Economic Problems (1943).
    10. Harris, S. E., Economic Reconstruction (1945).
    11. Hayes, H. Gordon, Spending, Saving and Employment (1945).
    12. League of Nations: Anti-Depression Policy (1945).
    13. Langum, John K., Postwar Banking Problems (1946).
    14. Postwar Economic Studies No. 3, Public Finance and Full Employment (1945).
    15. Postwar Economic Studies No. 8, Federal Reserve Policy (1946).
    16. Ruml and Sonne, Fiscal and Monetary Policy (1944).
    17. Terborgh, George, The Bogey of Economic Maturity (1945).
    18. Williams, John H. Postwar Monetary Plans (Second Edition, 1945), Chapters 4, 5.

Source: Harvard University Archives. Syllabi, course outlines and reading lists in Economics, 1895-2003 (HUC 8522.2.1) Box 4, Folder “Economics, 1947-48 (2 of 2)”.

____________________________

Year-end Exam

1947-48
HARVARD UNIVERSITY
ECONOMICS 141b
PRINCIPLES OF MONEY AND BANKING

(Three hours)

Discuss one question in each part.

I

  1. Your own appraisal of Keynes’ “General Theory.”
  2. The role of money in Keynes’ “General Theory”.

II

  1. Postwar Federal reserve policy.
  2. The secondary (government security) reserve proposal.

III

  1. International monetary and trade adjustment in the postwar world.
  2. Harrod’s “Are These Hardships Necessary?”
  3. The franc devaluation.

 

Final. May, 1948.

Source: Harvard University Archives. Harvard University Final Examinations 1853-2001. Box 14. Papers Printed for Final Examinations: History, History of Religions…, Economics, … , Military Science, Naval Science, May, 1947.

____________________________

 ECONOMICS 141
PRINCIPLES OF MONEY AND BANKING
GENERAL REFERENCE READING
[13 pages!]

Has been transcribed and posted with the material for 1946-47.

Source: Harvard University Archives. Syllabi, course outlines and reading lists in Economics, 1895-2003 (HUC 8522.2.1) Box 4, Folder “Economics, 1946-47 (2 of 2)”.

Image Source: Alvin H. Hansen and John H. Williams in Harvard Class Album 1942.

 

 

 

 

Categories
Exam Questions Johns Hopkins Suggested Reading Syllabus

Johns Hopkins. Income Distribution Theory, Readings and Exams. Machlup, 1950’s

 

 

The following reading list on the theory of income distribution taught by Fritz Machlup in the mid-1950s at Johns Hopkins University was found in a file in the Evsey Domar papers marked “Macroeconomics, Old Reading Lists”. I hadn’t realized until this post that Machlup’s papers are archived at the Hoover Institution, where 45 boxes alone are filled with the archival remains of his academic career. OK, next time.

I remember that my dissertation supervisor, the same Evsey Domar, did not particularly “like” Fritz Machlup. The two of them were at Johns Hopkins in the 1950s, Machlup being a dozen years Domar’s senior. It is not that Evsey Domar would have actually trash-talked Fritz Machlup in front of a student of his, but I do have a vague recollection of Domar judging Machlup’s approach to economics as having been excessively concerned with terminological issues over substantive economics. Also I sensed that Domar considered Machlup to have viewed matters of academic rank and relative status with excessive seriousness. But these memories fall closer to the legend end of the historical spectrum than to those frequencies reserved for documented anecdotes. 

____________________

JOHNS HOPKINS UNIVERSITY
THE THEORY OF RELATIVE INCOMES
18-603, Fall Term 1954-55
Prof. Fritz Machlup

READING LIST

Texts:

  1. American Economic Association, Readings in the Theory of Income Distribution. (Philadelphia: Blakiston, 1946)
  2. Any one of the books on the list below.

 

  1. General Background

Alfred Marshall, Principles of Economics (London: Macmillan, 8th ed. 1936) Books V and VI.

[Handwritten note, “theory of derived demand exp. Ch. 1-6”, apparently referring to Marshall, Book V (“derived demand” found in Chapter 6 of Book V)]

Eugen v. Böhm-Bawerk, Positive Theory of Capital (London: 1891; Reprinted New York, Stechert, 1940) Book III, Ch. X; Book IV, Ch. VII.

Philip H. Wicksteed, The Common Sense of Political Economy. London: Routledge, 1933) Vol. I, Book I, Chapter IX.

Frank H. Knight, Risk, Uncertainty and Profit (Boston: 1921, Repreinted London School of Economic) Part II.

John R. Hicks, Value and Capital (Oxford: Clarendon Press, 1939) Part II.

 

  1. General Equilibrium Theory

Gustav Cassel, A Theory of Social Economy (New York: Harcourt, Brace, 1924) Chapter IV.

Bertil Ohlin, Interregional and International Trade (Cambridge: Harvard Univ. Press, 1933) Appendix I.

George Stigler, Production and Distribution Theories (New York: Macmillan, 1941) Chapter IX and XII.

Joan Robinson, “Euler’s Theorem and the Problem of Distribution” Economic Journal, Vol. XLIV (1934).

 

  1. Marginal Productivity and Substitution

John Bates Clark, The Distribution of Wealth (New York: Macmillan, 1900) Chapter XII and XIII.

Joan Robinson, Economics of Imperfect Competition (London: Macmillan, 1934) Books VII, VIII, IX.

John R. Hicks, The Theory of Wages (London: Macmillan, 1935) Chapter I and VI.

Paul H. Douglas, The Theory of Wages (New York: Macmillan, 1934) Chapter III.

Paul H. Douglas, “Are There Laws of Production?” American Economic Review, Vol. XXXVIII (1948).

Fritz Machlup, “The Commonsense of the Elasticity of Substitution,” Review of Economic Studies, Vol. II (1935).

Richard A. Lester, “Shortcomings of Marginal Analysis for Wage-Employment Problems.” American Economic Review, Vol. XXXVI (1946)

Fritz Machlup, “Marginal Analysis and Empirical Research” American Economic Review, Vol. XXXVI (1946).

Articles by Cassels, Stigler, Chamberlin, Machlup, Robinson, Lange, and Kalecki in A.E.A. Readings.

 

  1. Wage

John R. Hicks, The Theory of Wages Chapters II, III, IV.

Paul H. Douglas, The Theory of Wages Chapter X.

Edwin Cannan, “The Demand for Labour”, Economic Journal, Vol. XLII. (1932)

Fritz Machlup, The Political Economy of Monopoly (Baltimore: Johns Hopkins, 1952) Chapters IX and X.

Articles by Robertson, Robbins, Bloom, Rolph, Reynolds, Lerner, Tarshis, and Dunlop in AEA Readings.

 

  1. Rent

David Ricardo, Principles of Political Economy and Taxation (1st ed. 1817) Chapter II.

Hubert D. Henderson, Supply and Demand (Cambridge: University Press, 1922, Revised, 1932) Chapter VI.

Joan Robinson, Economics of Imperfect Competition, Chapter VIII.

Gordon F. Bloom, “Technical Progress, Costs, and Rent”. Economica IX, New Series (1942)

Articles by Buchanan, and Boulding in AEA Readings.

 

  1. Interest

Eugen v. Böhm-Bawerk, The Positive Theory of Capital, Books II, V, VI, and VII.

Knut Wicksell, Lectures on Political Economy (New York: Macmillan, 1934) Vol. I, Part II, Ch. 2.

John Maynard Keynes, The General Theory of Employment, Interest and Money (London: Macmillan, 1936) Chapters 11, 12, 13 and 14.

Friedrich A. Hayek, The Pure theory of Capital (London: Macmillan, 1941) Chapters III, V, VI, VIII, XI-XIV.

Fritz Machlup, “Professor Knight and the ‘Period of Production’”, Journal of Political Economy, Vol. XLIII (1935).

____________ “The Rate of Interest as Cost Factor and as Capitalization Factor”, American Economic Review, Vol. XXV, (1935)

Articles by Hayek, Knight, Keynes, Robertson, Hicks, Somers, and Lutz, in Readings.

 

  1. Profit

Frank H. Knight, Risk, Uncertainty and Profit, Chapters IX-XII.

Joseph Schumpeter, The Theory of Economic Development (Cambridge: Harvard University Press, 1934) Chapter IV.

Robert Triffin, Monopolistic Competition and General Equilibrium Theory (Cambridge: Harvard University Press, 1940) Chapter V.

Fritz Machlup, The Economics of Sellers’ Competition (Baltimore: Johns Hopkins, 1952), Chapters VII and VIII.

Articles by Knight, Hart, Gordon, and Crum, in Readings.

Source: Duke University. David M. Rubenstein Rare Book and Manuscript Library. Economists’ Papers Project, Papers of Evsey Domar, Box 15, Folder “Macroeconomics, Old Reading Lists”.

____________________

THE JOHNS HOPKINS UNIVERSITY
The Theory of Relative Incomes
18-603

January 21, 1953

Professor Fritz Machlup

Answer three questions, one from each group.

Write on loose sheets of paper; start a new sheet for each question.
Identify each sheet by the Question Number in the left corner and your Examination Number (which you draw before the examination) in the right corner; your name should appear nowhere.(I.

  1. Describe in words, without using any symbols, the Walrasian system of general equilibrium, stating the essential assumptions, the variables assumed to be given, and the unknowns to be derived.

II.

  1. Discuss the influence of different types of inventions on the marginal productivity of labor. Indicate also their probably effects on the total income of the labor class and on its relative share in the national income.
  2. Dennis H. Robertson divides the effects which “an artificial raising of the wages” is apt to have upon employment into “two analytically separable reactions”, first, “a movement along the existing [marginal productivity] curve,” and second, “a cumulative lowering of the curve”. Explain the two reactions and indicate what assumptions concerning other factors of production, especially capital, are involved.

III.

  1. State the three grounds on which Böhm-Bawerk bases his explanation of the existence of interest and discuss whether each or any of them constitutes a necessary and/or sufficient condition of the existence of interest. (You may avoid committing yourself to the arguments expressed by attributing them to “some writers”.)
  2. Without indicating your own opinions or inclinations, present both sides in the controversy between Frank H. Knight and the “Austrians” with respect to the following points:
    1. that all capital is conceptually perpetual or conceptually non-permanent;
    2. that economic progress may result in a “shortening” of the investment period;
    3. that an increase in the supply of capital need not change the original factors of the remote past.

Source: Johns Hopkins University. Eisenhower Library, Ferdinand Hamburger, Jr. Archives. Department of Political Economy, Series 6, Exams, 1956-62. Box 3/1, Folder “Graduate Exams, 1933-1965”.

____________________

THE JOHNS HOPKINS UNIVERSITY
THEORY OF RELATIVE INCOMES
18.603

January 1957

Professor Fritz Machlup

Answer four questions, one from each part.

Write on loose sheets of paper; start a new sheet for each question.
Identify each sheet by the Question Number in the left corner and your Examination Number (which you draw before the examination) in the right corner; your name should appear nowhere.
You are on your honor not to use notes or to give or accept advice.

PART I.

  1. A product, X, is made from three “ingredients” or factors of production, A, B, and C, all of which are necessary and can be used only in a fixed proportion. Total output of X is 1000 units per unit of time; the product sells at a price of $100 per unit. The factor costs per unit of product are $60 for A, $30 for B, and $10 for C. The supplies of A and B are perfectly elastic to the industry. The demand for X has an elasticity of -2. The industry is competitive both in its buying and selling.
    Assume that the quantity of C which is available to the industry is reduced by 20 per cent. Calculate the elasticity of the industry’s derived demand for C. Show your reasoning step by step.
  2.      a. Define or explain the concept of elasticity of substitution as it is used by Mrs. Robinson.
    1. Is it “technical” substitution or “total” substitution which is involved in Mrs. Robinson’s concept? What is the difference between the two substitutabilities?

PART II.

  1. Discuss various concepts of “bargaining power” in the labor market, commenting on the selection of criteria, the problem of measurability, and the uses to which the concepts are put.
  2. Ricardo says in the chapter “On Rent” of his Principles of Political Economy and Taxation: “If the high price of corn were the effect, and not the cause of rent, price would be proportionately influenced as rents were high or low, and rent would be a component part of price. But that corn which is produced by the greatest quantity of labor is the regulator of the price of corn; and rent does not and cannot enter in the least degree as a component part of its price.” Discuss. Take account of the possibility that land has other uses besides the production of corn.

PART III.

  1. Without indicating your own opinions or inclinations, present both sides in the controversy between Frank H. Knight and the “Austrians” with respect to the following points:
    1. that all capital is conceptually perpetual or conceptually non-permanent;
    2. that economic progress may result in a “shortening” of the investment period;
    3. that an increase in the supply of capital need not change the original factors of the remote past.
    4. that it is not possible to identify the contributions of the original factors of the remote past.
  2. On p. 208 of his Lectures, Vol. I, Wicksell quotes the following statement by Gustav Cassel: “A man who attaches the same importance to future needs as to present ones, if he expects to be able to provide for his needs in the future just as easily as he does now, has no reason for setting aside anything of his present income.” According to Wicksell, “Cassel is not quite correct” inasmuch as his “argument actually presupposes the absence of any rate of interest.” Explain.

PART IV.

  1. What, if anything, does general-equilibrium theory contribute to the understanding or development of income-distribution theory?
    In order to facilitate a thoughtful discussion of this question it is suggested that you treat it in three parts:

    1. The function of a theory of relative incomes. (What is it designed to do? What kind of general principles or conceptual schemes seem to be useful in developing a theory of income distribution?)
    2. The essentials of general-equilibrium theory. (What is it designed to do and how? What do we learn from it?)
    3. The contribution, or lack of it, of general-equilibrium systems to the theory of relative incomes.

 

Source: Johns Hopkins University. Eisenhower Library, Ferdinand Hamburger, Jr. Archives. Department of Political Economy, Series 6, Exams, 1956-62. Box 3/1, Folder “Graduate Exams, 1933-1965”.

Image Source:  Fritz Machlup page  at the website Austrian Economics Center.

 

Categories
Johns Hopkins Seminar Speakers

Johns Hopkins. Economic Seminary, 1922-23

 

An earlier post on the economic seminary of Johns Hopkins University (1903-04) was taken from an official publication of the university. In the meantime I have found the original minutes of the “economic conference”, followed by the “economic seminary” for the period 1892-1951! Up through the 1921-22 academic year, the minutes are handwritten, reasonably legible, but still harder to read than the typed minutes beginning in 1922-23. I’ll start with the easier reading first.

The economic seminary schedule for the following years have also been posted:

1903-1904
1904-1905

1922-1923
1923-1924
1924-1925
1925-1926
1926-1927

__________________

POLITICAL ECONOMY
GRADUATE COURSES

The graduate instruction in Political Economy is designed primarily to meet the needs of advanced students preparing for a professional career in economic science. The courses afford systematic instruction in general economic principles, intimate acquaintance with special fields of economic activity, and, most important of all, knowledge of and ability to employ sound methods of economic research. The work centres in the Economic Seminary, the membership of which is limited to the most advanced students, and the primary design of which is to develop scientific research in economic study and investigation…

…The Economic Seminary

Two hours weekly through the year. Professors Hollander and Barnett, Associate Professor Weyforth, Miss Jacobs, and Dr. Mitchell.
The work of the year will be the study of representative forms of industrial development in the United States, and the analysis of significant activities of American labor organizations.

 

Source: The Johns Hopkins University Circular, Vol. XLI, 1922, p. 342.

__________________

MEMBERS OF THE ECONOMIC SEMINARY
1922-1923

Students

Black, Stanley Roberts

A.B., Colby College, 1921. 1st year, Political Economy, History, and Political Science.

Chen, Chao Ming

Fuhkien Privincial College. 2nd year undergraduate residence, candidate for A.B., Johns Hopkins University. Professor Latane, adviser.

Culver, Lydia Margaretta (Miss)

A.B. Goucher College, 1921. 1st year, Political Economy

Evans, George Heberton, Jr.

A.B., Johns Hopkins University, 1920. 2nd year, Political Economy, Political Science, and Psychology

Fedder, Abraham

A.B., Johns Hopkins University, 1921. 1st year, Political Economy, Political Science, and History

Fossum, Paul Robert

A.B. Park Region Luther College, 1915. 2nd year, Political Economy

Gillies, Robert Carlyle

A.B., Princeton University, 1920. 1st year (part-time), Political Economy

Hankin, Anne E.

Attendant on courses in social economics [1923]

Hartin, William McCants

B.Litt., Furman University, 1897, and A.M., 1899; Th.M., Southern Baptist Theological Seminary, 1904. Part-time, Political Economy

Howard, Mary Cushing (Miss)

Bryn Mawr School. Attendant on Courses in Economics and Business Economics.

Kahn, Hortense Miller

A.B., Goucher College, 1916. 1st year (part-time), Political Economy.

Levin, Benjamin Szold

Park School. Senior undergraduate, Dr. Weyforth, Adviser

Meeth, Ruth Elizabeth (Miss)

A.B., Goucher College, 1918. 1st year, Political Economy

Nelson, Louise Dallam

Attendant on courses in social economics.

Pasternak, Lillian

A.B., Goucher College, 1920. 1st year, Political Economy

Saiontz, Leon Robert

A.B., Johns Hopkins University, 1921. 2nd year, Political Economy

Suzuki, Hitoshi

Master of Commerce, Meiji University (Japan), 1920. 1st year, Political Economy

Tingley, Ruth (Miss)

A.B., Goucher College, 1914. 1st year, Political Economy

Whistler, Margaret Kathryn (Miss)

A.B., Goucher College, 1921. 1st year, Political Economy.

Wyckoff, Vertrees Judson

A.B., Princeton University, 1920. 1st year (part-time), Political Economy

Griffiss, Bartow [Added to list of members]

A.B., Johns Hopkins University, 1920. 2nd year, Political Economy, Political Science, and Psychology

Seibert , Louise Cleret (Mrs.) [Presented, but not on list of members]

A.B. Goucher College, 1920. (probably) 1st year (part-time) Political Economy

Taketomi, Yasuo [Presented, but not on list of members]

A.B., Waseda University. 2nd year, Political Economy

 

Faculty

Jacob H. Hollander, Professor of Political Economy [at present rank, 1904-; first appointment, 1894]

A.B., Johns Hopkins University, 1891, Fellow, 1893-94, and Ph.D., 1894; Treasurer of the Island of Porto Rico, 1900-01; Special Commissioner Plenipotentiary to Santo Domingo, 1905-06; Financial Adviser of the Dominican Republic, 1908-10.

George Ernest Barnett, Professor of Statistics [at present rank, 1911-; first appointment, 1901]

A.B., Randolph Macon College, 1891; Fellow, Johns Hopkins University, 1899-1900, and Ph.D., 1901.

William Oswald Weyforth, Associate (Associate Professor Elect) in Political Economy [at present rank, 1919-]

A.B., Johns Hopkins University, 1912, and Ph.D., 1915; Instructor, Western Reserve University, 1915-17.

Broadus Mitchell, Instructor (Associate Elect) in Political Economy [at present rank, 1919-]

A.B., University of South Carolina, 1913; Fellow, Johns Hopkins University, 1916-17, and Ph.D., 1918.

Miss Theo Jacobs, Associate in Social Economics [1919-]

A.B., Goucher College, 1901; Federated Charities of Baltimore (District Assistant, 1905-07, District Secretary, 1907-10, Assistant General Secretary, 1910-17, Acting General Secretary, 1917-1919.

__________________

 Minutes of the Seminary
1923-24

October 11, 1922

The opening meeting of the economic seminary of the session was held in room 315 on Wednesday at 2 o’clock. Mr. [Bartow] Griffiss read a paper on “The Relation Between Fluctuations in the Call Money Rate and in Stocks”.

October 18, 1922

The economic seminary met at 2 o’clock. Professor [George E.] Barnett read a paper on “Index Numbers of the Total Cost of Living as a Measure of Satisfaction”.

October 25, 1922.

Mr. [Robert Carlyle] Gillies read two papers , (1) “Minimum Health and Decency Budgets as Employed in Labor Union Arguments,” and (2) “Relative Cost of Government and Private Borrowing”.

November 1, 1922

Mr. [Vertrees Judson] Wyckoff read a paper on “Illustrations of the Tactics of Two Trade Unions During Period of Commercial Depression”.

November 8, 1922

Mr. Paul Robert Fossum read a paper on “The Agrarian Movement in North Dakota”.

November 15, 1922

Professor [Jacob H.] Hollander read a paper on “The Allied Debt”.

November 22, 1922

Mr. [Stanley Roberts] Black read a paper on “Recent Investment Policies of Mutual Savings Banks”.

November 29, 1922

Mr. [Leon Robert] Saiontz read a paper on “Tax Exempt Securities in the United States”.

December 6, 1922

Mr. [Abraham] Fedder read a paper on “History of the Index Numbers of the Cost of Living in Great Britain”.

December 13, 1922

Mr. [William McCants] Hartin read a paper on “Control of the Boll Weevil in the United States”.

December 20, 1922

Miss [Lillian] Pasternak read a paper on “Interrelated Dependent Families”.

January 3, 1923

Mr. [Bartow] Griffiss read a paper on “Control of the Call Money Rate”.

January 10, 1923

Mr. [Benjamin Szold] Levin read a paper on “The Farm Loan Act”.

January 17, 1923

Mr. [Robert Carlyle] Gillies read a paper on “Performance of the U.S. Railroad Administration”.

January 24, 1923

Mrs. [Louise Cleret] Seibert read a paper on “Unions of Government Employees”.

January 31, 1923

Miss [Mary Cushing] Howard read a paper on “The American Federation of Labor”.

February 7, 1923

Dr. [William O.] Weyforth read a paper on “Exchange Rates and Purchasing Power Parities”.

February 14, 1923

Professor [Jacob H.] Hollander gave a talk and read some Letters of Adam Smith.

February 21, 1923

Dr. [Broadus] Mitchell talked on “The Recent Accession to the Hutzler Collection”.

February 28, 1923

Mr. [Bartow] Griffiss talked on “The Operation of the New York Call Money Market”.

March 7, 1923

Mr. [Vertrees Judson] Wyckoff read a paper on the “Conclusions on the Readjustment of Trade Union Agreements in a Period of Depression”.

March 14, 1923

Mr. [Stanley Roberts] Black read a paper on “Legislative Policies in Respect to Investments of Mutual Savings Banks”.

March 21, 1923

Mr. [Paul Robert] Fossum read a paper on “The Functions of the Bank of North Dakota in the Industrial Program of the Non-Partisan League”.

March 28, 1923

Mr. [Hitoshi] Suzuki read a paper on “Post-War Finance in Japan”.

April 11, 1923

Mr. [Abraham] Fedder read a paper on “Occupational Changes Due to Invention and Improvements”.

April 18, 1923

Mr. [Leon Robert] Saiontz read a paper on “History of Tax Exempt Securities”.

April 25, 1923

Miss [Ruth Elizabeth] Meeth read a paper on “Placing Out Work of the Henry Watson Children’s Aid Society”.

May 2, 1923

Miss [Ruth] Tingley read a paper on “The Unmarried Mother and Her Child”.

May 9, 1923

Miss [Lydia Margaretta] Culver read a paper on the “History of Baltimore Association for the Improvement of the Conditions of the Poor”.

May 15, 1923

Mr. [Yasuo] Taketomi read a paper on “Thomas Hood’s Social Verse”.

May 23, 1923

Miss [Margaret Kathryn] Whistler read a paper on “Probation in Baltimore”.

Seminary ended for the year.

 

Sources:  

Dates, presenters and topics from: Johns Hopkins University. Eisenhower Library, Ferdinand Hamburger, Jr. Archives. Department of Political Economy, Series 1. Minutes of the Economic Seminary, 1892-1951. Folder “1922-1940”.

 Information about the members of the seminary from: The Johns Hopkins University Circular, Vol. XLI, 1922 and The Johns Hopkins University Circular, Vol. XLII, 1923.

Image Source:Webpage “Gilman Hall circa 1920” in the Hopkins Perspective, 1876-Today collection.

 

Categories
Business School Columbia Dartmouth Harvard Pennsylvania

Columbia School of Business Opens. Seligman’s Thoughts, 1916

 

Columbia University economist provides “the history of the movement which has culmination in the adoption of this project”, i.e. the founding of Columbia School of Business. The earlier resistence of the economics department to a School of Business is explained as well as the flip-flop to its support of opening of the School of Business in the autumn term of 1916.

_______________

A UNIVERSITY SCHOOL OF BUSINESS
by Edwin R. A. Seligman

[I]

THE opening of the Columbia School of Business in the autumn of 1916 marks another milestone in university education. The history of the movement which has culminated in the adoption of this project is highly interesting.

Less than a generation ago the only opportunities offered in education for business were the classes in single and double bookkeeping, usually conducted both here and abroad under the high- sounding title of “Business Institutes.” All they did was to give a smattering of ordinary bookkeeping with occasionally some slight instruction in English or a foreign language thrown in. One or two farsighted men already at that early period appreciated the need of a more systematic preparation for business life; but theirs were voices crying in the wilderness. It was the time when any kind of institutional education, except for the ministry, counted but little, the time when the lawyer was supposed to prepare himself for his work by serving an apprenticeship in a law office, and when the college graduate desirous of entering business life was at a disadvantage in the estimation of the employer as compared with the youth who had started from the bottom and who had enjoyed a few years of business experience. One of the broad-minded exceptions was Mr. Joseph Wharton of Philadelphia, through whose liberality the Wharton School was created at the University of Pennsylvania in the early eighties. This school, however, had at first only a moderate success, as did the similar schools started from time to time by other colleges and universities. The time was not yet ripe. When Columbia came to consider the problem, it preferred to devote its energies to political science rather than to business, and to purely University or graduate rather than to undergraduate work. As a consequence there was initiated the School of Political Science, which on its pedagogical side became a training school for teachers of the social sciences and for governmental administrators.

In the meantime, the economic development of the United States as well as of Europe led to a constant broadening of the scale on which business enterprises were carried on, and the demand for really adequate commercial training became more and more insistent. Toward the end of the last century the interest thus awakened became so strong that the Chamber of Commerce of New York was ready to grant an annual subvention to Columbia if it should be decided to develop courses of the desired character. The situation was canvassed by a small committee; but it was finally decided not to accept the overtures made by the committee of the Chamber of Commerce for several reasons. In the first place, it was felt that the demand had not yet become sufficiently great to justify the expectation of a student body satisfactory in either quantity or quality. Secondly, we were convinced that a successful school of the character desired would have to be conducted along academic lines of a modified kind, and that the best results could be hoped for only by securing academic teachers with a business experience rather than business men without academic experience. It was, however, at the time impossible to find a sufficient number of qualified instructors. Moreover, the literature of the subject was as yet embryonic, and the proper curriculum of such a School had nowhere been thoroughly worked out. In the third place, it was realized that the most important consideration at the time in American educational development, and especially at Columbia, was to emphasize the purely scientific or graduate work in political science; and the Department of Economics feared lest there might be danger in diverting its energies from the scientific field to work of a technical or professional character, such as would be necessitated by a new School of the kind contemplated. Finally, the movement for the creation of commercial high schools had come to a head, and it was deemed wiser to ascertain how far the gap might be filled by the secondary schools before deciding as to what should be done by Columbia. For these reasons the project was postponed, and the entire energy of the Department of Economics was directed to the rounding out of the University courses in political science and to the improving and broadening of the tender of the undergraduate or college course in economics.

During the last fifteen years, however, an instructive development has occurred. In the first place, there was a growing recognition of the need for a broader and more adequate training for business. Chambers of Commerce and other commercial bodies both here and abroad began to grow more restless and more insistent in their demands. The old feeling of prejudice on the part of the successful business man toward the college graduate diminished, although he still maintained that the college curriculum might profitably be modified in some respects to give a better preparation for business. This demand, which emanated primarily from the commercial community, now found expression in the new commercial schools in England and even more in Germany, and a rich fund of knowledge was being accumulated from the experience of these foreign schools. In the United States, moreover, it was gradually recognized that the commercial high schools, however excellently managed, were not quite adequate to solve the problem.

In the course of time professional schools of the desired kind were initiated, although along widely varying lines, by several American universities, the most notable examples being those of New York University and of Harvard. In New York City the demand for the inception of courses of some kind at Columbia soon became so urgent that a modest beginning was made three or four years ago with a few evening courses. Owing to the high standards which were observed from the outset, these courses met with immediate success. They were conservatively increased from year to year, until during the past year the number of students and the character of the instructors became such as to justify the demand for their merger into a new and independent school, which should possess an identity of its own and which should become a regularly accredited part of the University.

There were several reasons which led the Department of Economics now to welcome the movement to which it had been lukewarm a decade or two before. In the first place, the number of men qualified to serve as instructors in the new schools had become so numerous as to make it reasonably certain that the faculty could be filled by men of the first rank. Secondly, the literature of the subject had become so abundant as to make it possible to put academic teaching in business on a par with that of the other occupations or professions. Thirdly, experience with various types of schools had become so rich as to permit of what seemed to be a sound conclusion. Finally, the University work under the Faculty of Political Science had become so thoroughly established that there was no danger to be anticipated in any diversion of energy to the new institution. It was felt, therefore, that we were now quite ready to develop the technical or professional, rather than the purely scientific, sides of instruction in Economics.

It was for these reasons that the Department of Economics as well as the entire Faculty of Political Science cordially welcomed the project for the new School and that the report of the special committee appointed to consider the subject met with the unanimous approval of the University Council and was speedily adopted by the Board of Trustees.

II

In determining upon the character of the School, the committee considered with some care the different types in existence. There are in the United States at present three chief types: (1) the Wharton School, which has a curriculum of four years parallel to that of the college and which is essentially an undergraduate school; (2) the Harvard School of Business Administration, which has a two- years’ curriculum of a frankly graduate character; and (3) the Amos Tuck School at Dartmouth, which admits students at the end of the junior year and carries them through a two-years course. No one of these types approved itself to the committee.

The Wharton School plan seemed to be open to criticism from several points of view. As a purely undergraduate school it necessarily becomes a rival to the college and to the extent that it succeeds, it is likely to weaken the college. In the second place, it begins professional or technical work at too early a period, whereas the whole tendency of recent development in the United States is to relegate the professional or technical education to a somewhat later stage. The change that has been going on during the last few years in the Engineering Schools and other Schools of Applied Science affords ample evidence of this tendency. What is needed in this country is a broad foundation for the technical or professional class, and the School of Business needs as broad a foundation as we are coming to demand for other professional schools. Thirdly, a purely undergraduate school of business excludes the possibility of any pronounced extension of the graduate or research courses, which are coming to be as important in applied economics as they are in pure economics. A four-years’ undergraduate curriculum in business courses virtually exhausts the subject and leaves practically nothing for the research student. It was largely for these reasons that the Wharton School type was discarded as a model.

On the other hand the Harvard type seemed to be open to criticism for opposite reasons. In the first place, the requirement of a college degree for entrance renders such a school impotent to serve the public which is clamoring for admission in large centers like New York. Comparatively few men who intend to go into business can afford, whether from the material or from any other point of view, to wait until they are twenty-four or twenty-five years of age before entering upon a practical business career. And it is questionable whether even a few captains of industry will be recruited from this class. A purely graduate school which can never expect more than a handful of students is thus abandoning its opportunity to serve the public in the largest measure. In the second place, not only must such a school from the very nature of the case be numerically insignificant, but it seems to be based upon an erroneous pedagogical principle. It is now rather widely recognized that the movement inaugurated by President Eliot a generation ago went too far for the best interests of American education. In attempting to convert the American college into a university, he ignored the fact that the principles of academic freedom—freedom of the student as well as freedom of the teacher—are applicable in full measure only to a real university doing advanced or research work. Moreover, although by pulling up, as he thought, the American college, to a higher or university level, he advanced the age of graduation to about twenty-two, he at the same time made the attainment of the college degree a prerequisite to professional or research work. The college thus came to occupy the contradictory position of a university and of something less than a university. The consequences soon disclosed themselves. As soon as the demands of the public for a better medical and legal preparation became imperious, the complications began; for the medical school course was gradually lengthened to five years, and the law school course to three years, with a possibility of soon becoming four years. To make, as was now done, entrance to the professional schools conditional upon a college degree therefore meant that the young lawyer could not begin his life’s work before the age of twenty-five or twenty-six and the young doctor before the age of twenty-seven or twenty-eight.

This is an intolerable situation, which exists nowhere else in the civilized world and which it is out of the question to think will permanently continue in the United States. The first step away from this difficulty was taken by Columbia some twenty years ago when it introduced the so-called combined course into the professional schools, permitting the saving of at least one year. This combined-course idea rapidly spread throughout the country and is now adopted by most of the leading universities, barring a few conservative institutions in the East. A slight modification of this system was later introduced at Columbia in the Schools of Engineering, Mining, and Chemistry, which were put upon a basis of advanced standing requiring three years of college work for entrance, thus making possible a combined course of six years from entrance into the college up to the acquirement of the professional degree. Even this, however, was gradually found to be inadequate; and before long not only the School of Medicine but the School of Architecture, and the School of Journalism opened professional courses to students who had completed two years of college work.

By many it was recognized that here is the proper dividing line between the ordinary cultural and preparatory courses on the one hand, and the technical or professional courses on the other. To those who hold to this opinion, it seems entirely probable that sooner or later the combined or Columbia plan, which has now spread throughout the country, will be replaced by the newer or still more distinctive Columbia plan, which is in harmony not only with the educational practice of the rest of the world, but with sound educational theory. The Harvard School of Business Administration, therefore, appeared to the committee to embody the same erroneous principle which had been applied to the law and medical schools. The country has broken away from the Harvard plan in legal and medical education. It seems unlikely that it will follow Harvard in the new form of business education. At all events, the system seemed to be quite inapplicable to conditions at Columbia.

The third type of business school is represented by the Amos Tuck School, which does, indeed, accept the principle of a dividing line below the close of the college curriculum. The Amos Tuck School, however, has turned out to be distinctly restricted in scope and attracts few students outside of Dartmouth itself. What it does is to provide an alternate year for Dartmouth seniors, with an opportunity of proceeding for an additional year. It does not succeed in drawing from other colleges students who have completed three years of college work. Moreover, it suffers from the same defect as the Harvard School in that it offers an inadequate curriculum of only two years in length.

Since therefore none of the existing types seemed to be either suitable to Columbia conditions or in harmony with sound pedagogical principles, it was decided to put the dividing line between college and professional work at the end of the second year, largely for the reasons mentioned above. Students will therefore be admitted to the Columbia School who have completed two years of college work or its equivalent, and the School of Business will be put on the same basis as the Medical School, the School of Architecture, and the School of Journalism. This arrangement makes possible the attainment of several results. In the first place, every student who enters the Business School as a candidate for a degree will be sure of having pursued those general cultural and disciplinary college courses which are considered obligatory upon every cultivated man in Europe as in America. In the second place, on this broad basis there will be erected a carefully devised professional or technical curriculum after the completion of which the graduate can enter upon his business career at the age of twenty-two or twenty-three,—about the ordinary age abroad. In the third place, the three-year course, which is midway between the exaggerated four-year Wharton course and the inadequate two-year Harvard and Amos Tuck courses, will permit a comprehensive and well-rounded sequence of studies. The type of school finally adopted thus seems to combine a maximum of advantages with a minimum of defects. It will moreover enable the School to serve much more varied classes of students than can be found in any other type.

Among these classes are, first, students who have spent two years in Columbia College or in some other college of equivalent rank and who are candidates for a degree. It is expected that not a few college students, both at Columbia and elsewhere, who have decided by the end of the second year to pursue a distinctively business career, will enter the new School and thus secure a better preparation for their life work than if they were to continue in a more or less desultory fashion through the remainder of their college career.

In the second place, the School will afford abundant opportunity in its upper reaches for graduate students who desire to prepare themselves for the teaching profession or who are inclined to devote time to purely research courses. Such students will be able to combine a more technical or professional course in the School of Business with graduate courses given in the School of Political Science, and there will therefore be offered for the first time in the United States a unique combination of pure and of applied science, or of theoretical and of practical economics, which will doubtless turn out to be fruitful of results.

In the third place, the School will afford an opportunity to graduates of high schools, who for some reason do not desire to go to college, to take courses in the Department of Extension Teaching at Columbia, in either day or evening courses, and to complete work equivalent to that offered by Columbia College in its first two years.

In the fourth place, there are in New York City many men and some women actively engaged in business who are eager to learn more about the real foundation of their business life. Students of this character, if over twenty-one years of age, who have shown their qualifications to undertake certain courses may be admitted as special students in particular subjects, but will, of course, not be candidates for a degree.

It is therefore believed that the type of school finally adopted is the one which will minister most successfully to the needs of the New York public, and which will, at the same time, provide on the broadest possible basis a curriculum which will attract students from all parts of the country.

III

Before we proceed to discuss the curriculum a word must be said about the name of the new institution. Most of the existing institutions are called Schools of Commerce or of Commercial Science. Such an appellation seemed, however, unsatisfactory. For in the first place what is taught in such a school is not primarily science at all, but art; or even if the purely scientific problems may be taken up in the later years of the School, the earlier years must naturally devote themselves primarily to the practical applications. But, more important than this, the term commerce seems to be ill-chosen. There are many problems of business management which have only a slight relation to commerce as such; and the Supreme Court of the United States has told us in a leading decision that insurance is not commerce at all. As in every School of this kind the problems connected with insurance must occupy a prominent place, it seems objectionable to apply a generic name in connection with a particular division to which the generic name is, as we are instructed, wholly inapplicable. On the other hand, some schools call themselves Schools of Business Administration. This title, however, is equally open to criticism. If we object to the term commercial science on the ground that a great part of the work is not science at all, we can equally object to the term business administration on the ground that a great part of the work far transcends purely administrative problems. What such a School has to deal with is the principles underlying business practice, as well as the best method of putting those principles into operation. It is partly science and partly administration; it is more than science and more than administration. Since, therefore, the real object of such a School is to deal with business problems in their varied and comprehensive aspects, it seemed wise to take the simple and obvious name of School of Business. In the Law School we study law; in the Medical School we study medicine; in the School of Architecture we study architecture; in the School of Engineering we study engineering; and consequently the obvious place in which to study business is the School of Business. The name is simple, inclusive, and comprehensive.

When we come to discuss the curriculum of the new School, several points are to be noted. In the first place, an attempt is made to steer between the rigid and fixed curriculum found in some of the American professional schools and the very elastic schemes that are found in the ordinary university courses here and abroad. It was attempted to strike a happy medium by requiring in the first year from all candidates for a degree a certain number of courses aggregating one-half or two-thirds of the whole. Every student who intends to go into business should know something about general economics, accounting, finance and business organization, and should also have a command of some of the foreign languages. When, however, the foundation has been laid in this way, students are allowed a free choice, subject to the condition, however, that their course be approved by the Director. The Director of the School is presumed to have a personal acquaintance with each of the students, and to be able in person or through delegation to give to each proper advice. Students who desire to have a general business course will find such a curriculum mapped out for them. Others who may prefer to specialize will find a sequence of courses in a variety of subjects: accounting, banking, finance, transportation, commerce and trade, business organization and management, manufactures, advertising and salesmanship, and the like. At the end of the second year, the degree of Bachelor of Science will be awarded so that those who do not care to defer their entrance into a practical business career may start in at the age of the ordinary college graduate. It is expected, however, that a large proportion of the students will continue for a third year, at the end of which the Master’s degree will be conferred.

This third year, it is hoped, will be the most valuable, as it will be the most unique, year in the School. It will correspond approximately to the clinical year which is now being added to our best medical schools. It goes without saying that in the City of New York, the centre of American wealth, the business problems are on a particularly gigantic scale and of a specially intricate character. It is proposed to make the courses in this third year not alone research courses in the more refined and difficult principles underlying business practice, but also practical courses where each student will have an opportunity of intimate personal contact with business life. Arrangements have already been made with the National City Bank whereby a certain number of students will be afforded an opportunity to prepare themselves for the service of the Bank in foreign fields. It is proposed to broaden and generalize these opportunities so that ultimately every student will be enabled and expected to do some field work in that particular department of business life in which he is especially interested. In almost every phase of “big business” in New York today the need is experienced for more expert and thorough training; and it is hoped in the advanced courses of the School to bring about a close cooperation between the corps of instructors on the one hand and the business community on the other. It is here that the School of Business will find an unexampled opportunity and perform an unexampled service. Just as the finest medical schools can exist only where there are the greatest hospitals, that is, in the large centres of population, so the most successful schools of business in the future may be expected to be found in the great centres of business life.

In order to accomplish these results and to realize the expectations which have been formed, it goes without saying that the new School of Business must be put on the highest possible standard of educational efficiency. So far as the students are concerned, this result has been guaranteed by the determination to make the scholastic standard as high as it is in the other departments of Columbia. We are fortunate in having in Dr. Egbert, as Director of the School, a man who is not only one of the great administrators in the country, but who has shown in both the Summer Session and the Extension work his adherence to these high standards. The continually growing reputation of those phases of the work to which Dr. Egbert has hitherto addressed himself are the surest guarantee of success in this new field.

High standards, however, depend not only upon the student body, but upon the corps of instructors. In order to avoid the difficulty which has unfortunately been experienced by so many American institutions, it is proposed that a professor must have one at least of two qualifications. If he is recruited from the academic ranks, he must possess the degree of Ph.D., to show that he has attained the highest academic honors, together with a reasonable business experience or an acquaintance with actual business problems. If, on the other hand, he is selected from the ranks of those who have devoted themselves primarily to business, he must not only have written a book which is an acknowledged authority in its field, but must give evidence of ability successfully to present the subject to the professional student. Although the corps of instructors is by no means entirely complete, it will be found that the selection has in every case been in accordance with the above considerations. The numerous additions to the teaching staff which are being planned for in the near future are confidently expected to conform to the same high principles.

Thus from every point of view, we feel that the problem has been carefully considered and solved with a reasonable hope of success. In the character of the student body, in the selection of the present and future teaching force, in the rounded sequence of courses, in the judicious union of practical and research work, in the rich possibilities of cooperation with the other departments of the University and the business life of the community, and last but not least, in the tried administrative experience of the Director, we have reason to believe that we possess a unique combination of factors which cannot fail to put the Columbia School of Business in the front rank of similar institutions here and abroad.

 

Source: Columbia University Quarterly, Volume XVIII, June 1916, pp. 241-252.

Image Source: From  American Economic Review, 1943.